Вы находитесь на странице: 1из 68

Graduate School of Health Science, Management and Pedagogy Southwestern University

MODULE REQUIREMENT FOR MIDTERM AND FINALS


MSN 504 I. POINT COUNTERPART The incidence of tuberculosis (TB) declined for 30 years after the introduction of antitubercular drugs. Unfortunately, tuberculosis reemerged as a serious threat to public health in the mid-1980s. The significant rise in tuberculosis can be attributed in part to the increased prevalence of human immunodeficiency virus (HIV) infections, but resurgence of TB has been documented in a variety of health care settings. Standard treatment regiments for tuberculosis involve self-administration of specific drugs for a period of several months. However, approximately one third of patients with active disease do not follow the prescribed treatment regimen. Their disease then relapses. Current protocols promoted directly observed therapy (DOT) as an effective alternative to the standard self-administration strategy. DOT requires that patients receive and swallow their drugs under supervision implementation of DOT has demonstrated a significant decrease in the frequency of primary drug resistance, as well as reductions in rates of acquired drug resistance and relapse. Management costs for DOT and standard antitubercular therapy are comparable. The costs for one patients who fails standard treatment regiments and develops multidrug-resistant tuberculosis far outweigh the cost of implementing DOT for several hundred patients.

1. What factors make self-administration of drugs difficult for patient?


2. What behaviors may contribute to primary and acquired drug resistance?

II. POINT COUNTERPART Cerebral palsy presents challenges to the patient, family, and health care provider. One of its cruelest manifestations is painful spasticity. The pain related to this element of the diseases easily qualifies as malignant, for its causes intense and long-term suffering while serving no useful physiologic purpose. The patient and all concerned already know that the disease is serious with poor prognosis. Early attempts to use baclofen, marketed as baclofen tablets, met with indifferent success. Concern about drug effects properly limited daily doses and approved durations of therapy. Even patients who were aided by baclofen therapy frequently could not sustain therapeutic levels because of the adverse effects at high dosages. Difficulties in swallowing large numbers of tablets were remedied only imperfectly by preparation of extemporaneous oral liquid formulations. Because baclofen is a centrally acting skeletal muscle relaxant, investigation of direct injection into the intrathecal space offered a reasonable focus for research. Results in a significant number of patients have indicated that baclofen administered via this route is of genuine value. A parenteral formulation without presevatives for intrathecal injection came to market as an orphan drug under federal Food and Drug Administration (FDA) guidelines. Therapy with intrathecal baclofen requires a trial of the drug by bolus injection. Continuous treatment requires surgical placement of a programmable infusion pump, which is refilled at intervals thereafter. Treatment exposes the patient to all the hazards of lumbar puncture, intrathecal catheter placement, and use of an implanted pump. Treatment is also quite expensive. Although the use of botulism toxin (Botox) is controversial, it is also being used in some children to paralyze specific muscles. Exposure to the botulism organism has been identified as creating unnecessary risks for other illness in a child whose health is already compromised. 1. Do you feel that a patient with spasticity of cerebral palsy must fall on oral baclofen therapy before alternative therapies can be considered for treatment? 2. What problems, if any, do you perceive from the fact that the company sponsoring baclofen as an intrathecal injection is also a leading manufacturer and marketer of implanted progmmable infusion pumps? 3. What reservations would you have in evaluating continuing education materials about intrathecal baclofen therapy of other experimental therapies? That were provided by the corporate sponsor of the drug? Would FDA requirements for such materials answer your reservations/

III. POINT-COUNTERPART The use of central nervous system stimulant to help control attention-deficit hyperactivity disorder (ADHD) and other behavior problems has become popular in recent years. The supporters of this practice say that these drugs are a relatively inexpensive and effective way to solve undesirable behavior problems. Supporters also say that science has provided us with a tool that is reasonably safe to use. The drugs do correct many problems quickly and avoid the cost of long-term therapy. In addition, the child gets immediate benefit of improved social relationships, academic achievement and behavior control, all of which increase a sense of self-esteem. Opponents of this practice say that the science of neurophysiology and neurochemistry, although progressive has a long way to go. Opponents believe that the use of psychoactive drugs, especially with young children, gambles that there will be no long-term adverse effects. Stimulant therapy is relatively new, and the drugs have not been used long enough to determine if there will be long-term consequences. Although it may not be as cost-effective in the short term to correct environmental circumstances that contribute to behavior problems, the long-term benefits may be much greater.

1. What is the economic cost to society of treating versus not treating attentiondeficit hyperactivity disorder?

2. Who should determine whether a child has attention-deficit hyperactivity disorder: the parents, health care provider, social worker, school system, or psychologist? 3. How should the child who is receiving a CNS stimulant be monitored? How long should therapy last? Are there other management modalities that would be equally effective?

Module in MSNG 201


for Alternative Instructional Module (AIM) Students

Odilon A. Maglasang,R.N.,M.A.N. Professor

Unit I Alterations in Oxygenation Module 1 The Cardiovascular System Overview Throughout the continuum of care, whether in a home, hospital, or rehabilitation setting, all patients with cardiovascular disease (disorders of the heart and major blood vessels; CVD) require similar assessments. Key components of the cardiovascular assessment include obtaining a health history, performing a physical assessment, and monitoring a variety of laboratory and diagnostic test results. An accurate and timely assessment of cardiovascular function provides the data necessary to identify nursing diagnoses, formulate a plan of care, and evaluate the response of the patient to the care provided. Essential to the development of these assessment skills is an understanding of the structure and function of the heart in health and in disease (Brunner and Suddarth, 2007). The dynamic circulatory system supplies oxygen and nutrients to the tissues of the body and removes and transports metabolic wastes to the excretory organs. Blood acts as the carrier. The heart pumps the blood through the arteries to the arterioles, which interconnect with the capillaries, venules and veins to return blood to the heart. The vessels carrying the blood

Objectives After you have completed this module, you should be able to: 1. Explain cardiac physiology in relation to cardiac anatomy and the conduction system of the heart 2. describe how the cardiac rate is regulated 3. explain the FrankStarling Law of the Heart 4. describe how contractile strength is influenced by epinephrine 5. discuss the factors that influence the return of venous blood to the heart 6. Compare central venous pressure monitoring, pulmonary artery pressure monitoring, and systemic intra-arterial

must remain intact and the pumping mechanism must maintain a dynamic action for continual movement of blood (Bullock and Henze, 2000). The basis for the pumping mechanism is the hearts cellular structure, which contracts and relaxes in a continual sequence to maintain blood flow (Bullock and Henze, 2000). Suggested Reading Activity Van De Graaf, Kent M. et. al.1997. Synopsis of Human Anatomy and Physiology. St. Louis, Missouri, USA: McGraw-Hill Companies, Inc. Chapter 15, pp. 371-423 Martini, Frederic H. 2002. Fundamentals of Anatomy and Physiology. 6th Edition. New York, New York: McGraw-Hill Companies, Inc.

The Conduction System Normal contraction of the heart is initiated by specialized conductive tissues, which are actually myocardial muscle cells with fewer myofibrils than the other myocardial cells. The conductive structures of the heart include: SA node, atrial internodal tracts, AV node, Bundle of His, right and left bundle branches, and terminal or Purkinje fibers. Suggested Reading Activity Van De Graaf, Kent M. et. al.1997. Synopsis of Human Anatomy and Physiology. St. Louis, Missouri, USA: McGraw-Hill Companies, Inc. Martini, Frederic H. 2002. Fundamentals of Anatomy and Physiology. 6th Edition. New York, New York: McGraw-Hill Companies, Inc.

Physiology of Cardiac Conduction

Cardiac electrical activity is the result of the movement of ions (charged particles such as sodium, potassium, and calcium) across the cell membrane. The electrical changes recorded within a single cell result in what is known as the cardiac action potential In the resting state, cardiac muscle cells are polarized, which means an electrical difference exists between the negatively charged inside and the positively charged outside of the cell membrane. As soon as an electrical impulse is initiated, cell membrane permeability changes and sodium moves rapidly into the cell, while potassium exits the cell. This ionic exchange begins depolarization (electrical activation of the cell), converting the internal charge of the cell to a positive one. Contraction of the myocardium follows depolarization. The interaction between changes in membrane voltage and muscle contraction is called electromechanical coupling. As one cardiac muscle cell is depolarized, it acts as a stimulus to its neighboring cell, causing it to depolarize. Sufficient depolarization of a single specialized conduction system cell results in depolarization and contraction of the entire myocardium (Brunner and Suddarth, 2007). Repolarization (return of the cell to its resting state) occurs as the cell returns to its baseline or resting state; this corresponds to relaxation of myocardial muscle. After the rapid influx of sodium into the cell during depolarization, the permeability of the cell membrane to calcium is changed. Calcium enters the cell and is released from intracellular calcium stores. The increase in calcium, which occurs during the plateau phase of repolarization, is much slower than that of sodium and continues for a longer period. Cardiac muscle, unlike skeletal or smooth muscle, has a prolonged refractory period during which it cannot be restimulated to contract. There are two phases of the refractory period, referred to as the absolute refractory period and the relative refractory period. The absolute refractory period is the time during which the heart cannot be restimulated to contract regardless of the strength of the electrical stimulus. This period corresponds with depolarization and the early part of repolarization (Brunner and Suddarth, 2007). The Action Potential An action potential (AP) is an electrical current generated by excitable tissues, in this case, the heart. When the myocardial muscle is at rest, the resting membrane potential is approximately -85 to -95 mV. When the AP occurs, the membrane potential changes from a negative to a slightly positive value, a process called depolarization.

The cardiac muscle cell remains depolarized for a longer period thando other excitable cells, which explains the plateau phase of the cardiac AP. The physiologic importance of this phase is that it allows for a longer contraction in cardiac muscle. Contraction begins during the plateau phase of the AP and reaches its peak of strength during early depolarization. Depolarization is followed by repolarization and a return to the resting state. These changes in potential and state of cardiac muscle can be described in terms of discrete phases: phase 0 denotes depolarization; phase 1 indicates complete depolarization; phase 2 is a plateau phase of maximum cardiac contraction; and phase 4 indicates the myocardium is at rest. Suggested Reading Activity Porth, Carol Mattson. 2008. Pathophysiology: Concepts of Altered Health States. 7th Edition. Philadelphia, USA: Lippincott Williams and Wilkins Bullock, Barbara L. and Henze, Reet L. 2000. Focus on Pathophysiology. Philadelphia, USA: Lippincott Williams & Wilkins.

Electrical Events of the Cardiac Cycle The cardiac cycle is initiated through specialized conduction tissue whose pacemaker cells spontaneously and rhythmically depolarize. The critical threshold is gradually reached; spontaneous depolarization occurs and is followed by repolarization. The cardiac cycle is divided into two major periods: systole, when the ventricles are contracting, and diastole, when the ventricles are relaxed. Suggested Reading Activity Martini, Frederic H. 2002. Fundamentals of Anatomy and Physiology. 6th Edition. New York, New York: McGraw-Hill Companies, Inc. Van De Graaf, Kent M. et. al. 1997. Synopsis of Human Anatomy and Physiology. St. Louis, Missouri, USA: McGraw-Hill Companies, Inc.

The Cardiac Output and the Cardiac Rate The amount of blood ejected by each ventricle of the heart is measured as the cardiac output (CO), which is equal to the volume of blood pumped per minute by each ventricle. The average resting cardiac rate in an adult is between 60-100 beats per minute; the average stroke volume (volume of blood pumped per beat by each ventricle) is 75 mL per beat. To calculate the CO, the two variables mentioned are multiplied, yielding an average CO of about 5.5 L/min but it varies greatly depending on the metabolic needs of the body.. The CO is linked to or blood volume. The total blood volume in a human body is also about 5.5 liters. This means that each ventricle pumps the equivalent of the total blood volume each minute under resting conditions. The Regulation of the Stroke Volume (SV) Principles that govern fluids and their movements influence cardiac activity and stroke volume. Before looking at cardiac activity, a few reminders are needed. First, remember anywhere fluids are present, they exert pressure against their container. Human body fluids are mostly water, so this pressure is referred to as hydrostatic pressure. When referring to blood, that same hydrostatic pressure is simplified to blood pressure. Second, as you know from experience, fluids always move from an area of high pressure to an area of low pressure. Lastly, when fluids moving through tubes, the friction of the molecules against the wall of the tube opposes, or resists the flow. This opposition is called frictional resistance, or simply resistance. As you have learned previously, the SV is the volume of blood pumped per beat by each ventricle. The stroke volume is regulated by the three variables:
1. end-diastolic volume (EDV), which is the volume of blood

in the ventricles at the end of the diastole;

2. total peripheral resistance (TPR), which is the frictional

resistance, or impedance, to blood flow in the arteries; and 3. contractility (C), or strength, of ventricular contraction. The SV is directly proportional to the EDV; an increase in EDV results in an increase in SV. The SV is also directly proportional to

contractility. Thus, when the ventricles contract more forcefully, they empty more completely and therefore, pump more blood. The reverse is also true. Control of Stroke Volume Stroke volume is primarily determined by three factors: preload, afterload, and contractility. Preload is the term used to describe the degree of stretch of the cardiac muscle fibers at the end of diastole. The end of diastole is the period when filling volume in the ventricles is the highest and the degree of stretch on the muscle fibers is the greatest. The volume of blood within the ventricle at the end of diastole determines preload. Preload has a direct effect on stroke volume. As the volume of blood returning to the heart increases, muscle fiber stretch also increases (increased preload), resulting in stronger contraction and a greater stroke volume. This relationship, called the Frank-Starling Law of the heart (or sometimes the Starling Law of the heart), is maintained until the physiologic limit of the muscle is reached. The Frank-Starling law is based on the fact that, within limits, the greater the initial length or stretch of the cardiac muscle cells (sarcomeres), the greater the degree of shortening that occurs (Brunner and Suddarth, 2007). The second determinant of stroke volume is afterload, the amount of resistance to ejection of blood from the ventricle. The resistance of the systemic BP to left ventricular ejection is called systemic vascular resistance. The resistance of the pulmonary BP to right ventricular ejection is called pulmonary vascular resistance. There is an inverse relationship between afterload and stroke volume. For example, afterload is increased by arterial vasoconstriction, which leads to decreased stroke volume. The opposite is true with arterial vasodilation: afterload is reduced because there is less resistance to ejection, and stroke volume increases (Brunner and Suddarth, 2007). Contractility is a term used to denote the force generated by the contracting myocardium under any given condition. Contractility is enhanced by circulating catecholamines, sympathetic neuronal activity, and certain medications (eg, digoxin, intravenous dopamine or dobutamine). Increased contractility results in increased stroke volume. Contractility is depressed by hypoxemia, acidosis, and certain medications (eg, beta-adrenergic blocking agents such as atenolol [Tenormin]). The heart can achieve a greatly increased stroke volume

(e.g., during exercise) by increasing preload (through increased venous return), increasing contractility (through sympathetic nervous system discharge), and decreasing afterload (through peripheral vasodilation with decreased aortic pressure) (Brunner and Suddarth, 2007).

Question No. 1 Describe how the cardiac output is affected by the cardiac rate and stroke volume. Control of Heart Rate Cardiac output must be responsive to changes in the metabolic demands of the tissues. For example, during exercise the total cardiac output may increase fourfold, to 20 L per minute. This increase is normally accomplished by approximate doubling of both the heart rate and the stroke volume. Changes in heart rate are accomplished by reflex controls mediated by the autonomic nervous system, including its sympathetic and parasympathetic divisions (Brunner and Suddarth, 2007). The parasympathetic impulses, which travel to the heart through the vagus nerve, can slow the cardiac rate, whereas sympathetic impulses increase it. These effects on heart rate result from action on the SA node, to either decrease or increase its inherent rate. The balance between these two reflex control systems normally determines the heart rate. The heart rate is stimulated also by an increased level of circulating catecholamines (secreted by the adrenal gland) and by excess thyroid hormone, which produces a catecholamine-like effect. Heart rate is also affected by central nervous system and baroreceptor activity (Brunner and Suddarth, 2007). Baroreceptors are specialized nerve cells located in the aortic arch and in both right and left internal carotid arteries (at the point of bifurcation from the common carotid arteries). The baroreceptors are sensitive to changes in blood pressure (BP). During elevations in BP (hypertension), these cells increase their rate of discharge, transmitting impulses to the medulla. This initiates parasympathetic

activity and inhibits sympathetic response, lowering the heart rate and the BP. The opposite is true during hypotension (low BP). Hypotension results in less baroreceptor stimulation, which prompts a decrease in parasympathetic inhibitory activity in the SA node, allowing for enhanced sympathetic activity. The resultant vasoconstriction and increased heart rate elevate the BP (Brunner and Suddarth, 2007).

Frank-Starling Law of the Heart The proportion of the EDV that is ejected depends upon the strength of ventricular contraction. Normally, contractile strength is sufficient to eject 70-80 mL of blood out of a total EDV of 110 to 130 mL. The ejection fraction is thus about 60%. More blood is pumped per beat as the EDV increases, and thus the ejection fraction remains relatively constant over a range of EDV. In order for this to be true, the strength of ventricular contraction must increase as the EDV increases. Experiments demonstrate that the strength of ventricular contraction varies directly with the EDV. Even in experiments where the heart is removed from the body (and is thus not subject to neural or hormonal regulation), and where the heart is filled with blood flowing from a reservoir, an increase in EDV within the physiological range results in increased contraction strength and, therefore, in increased stroke volume. This relationship between EDV, contractile strength, and SV is thus a built-in or intrinsic, property of the heart muscle, and is known as the Frank-Starling Law of the Heart (Van De Graaf, K., 1997). Question No. 2 Using the Frank-Starling Law, explain how the stroke volume is affected by: 1. bradycardia, and 2. a missed beat

Regulation of Blood Flow

Extrinsic Regulation. This refers to control by the autonomic nervous system and endocrine system. Activation of the sympathoadrenal system produces an increase in the CO, and thus an increase in the total blood flow through the body. Intrinsic Regulation. Intrinsic control mechanisms are classified as either myogenic or metabolic. Some organs, the brain and kidneys in particular, use these intrinsic mechanisms to maintain relatively constant flow rates in the face of the wide fluctuations in blood pressure (BP). This ability is termed as autoregulation. Suggested Reading Activity Van De Graaf, Kent M. et. al. 1997. Synopsis of Human Anatomy and Physiology. St. Louis, Missouri, USA: McGraw-Hill Companies, Inc. Martini, Frederic H. 2002. Fundamentals of Anatomy and Physiology. 6th Edition. New York, New York: McGraw-Hill Companies, Inc.

Question No. 3 Describe the mechanisms that increase blood flow to skeletal muscles during exercise.

Regulation of Blood Pressure Resistance of flow in the arterial system is greatest in the arterioles because these vessels have the smallest diameters. Blood flow rate and pressure are thus reduced in the capillaries, which are located downstream of the high resistance imposed by the arterioles. The BP and flow rate within the capillaries are further reduced by the fact that their total cross-sectional area is much greater, due to their larger number, than the cross-sectional areas of the arteries and arterioles. Baroreceptors are stretch receptors located in the aortic arch and in the carotid sinuses. An increase in pressure causes the walls of these arterial regions to stretch and stimulate the activity of sensory

nerve endings. A fall in pressure below the normal range, by contrast, reduces the frequency of action potentials produced by these sensory nerve fibers. Question No. 4 With reference to the Baroreceptor reflex, explain why a person who is dehydrated or who has lost a lot of blood has a rapid pulse and cold, clammy skin. Instructional References: Bullock, Barbara L. and Henze, Reet L. 2000. Focus on Pathophysiology. Philadelphia, USA: Lippincott Williams & Wilkins. Martini, Frederic H. 2002. Fundamentals of Anatomy and Physiology. 6th Edition. New York, New York: McGraw-Hill Companies, Inc. Porth, Carol Mattson. 2005. Pathophysiology Concepts of Altered Health States. Philadelphia, USA: Lippincott Williams and Wilkins Seeley, Rod R. et. al. 2006. Anatomy and Physiology. 7th Edition. New York, New York: McGraw-Hill Companies, Inc.

Unit I Alterations in Oxygenation Module 2 The Respiratory System Objectives After you have completed this module, you should be able to: 1. Describe the primary structures and functions of the respiratory system 2. Describe ventilation, perfusion, diffusion, shunting, and the relationship of pulmonary circulation to these processes. 3. Summarize the physical principles governing the diffusion of gases into and out of the blood 4. Explain the important structural features of the respiratory membrane 5. Describe the partial pressures of O2 and Overview Respiration provides the body with a means of gas exchange. It is the process whereby oxygen from the air is transferred to the blood and carbon dioxide is eliminated from the body. Respiration can be divided into three parts: ventilation, or the movement of air between the atmosphere and the respiratory portion of the lungs; perfusion, or the flow of blood through the lungs; and diffusion, or the transfer of gases between the air-filled spaces in the lungs and the blood. The nervous system controls the movement of the respiratory muscles and adjusts the rate of breathing so that it matches the needs of the body during various levels of activity. The content in this chapter focuses on the structure and function of the respiratory system as it relates to these aspects of respiration (Brunner and Suddarth, 2007).

Disorders of the respiratory system are common and are encountered by nurses in every setting from the community to the intensive care unit. To assess the respiratory system, the nurse must be skilled at differentiating normal assessment findings from abnormal ones. Good assessment skills must be developed and used when caring for patients with acute and chronic respiratory problems. In addition, an understanding of respiratory function and the significance of abnormal diagnostic test results is essential (Brunner and Suddarth, 2007). For purposes of this discussion, we will use two divisions of the circulatory system. One is pulmonary circuitry which sends deoxygenated blood to the lungs for removal of CO2 and uptake of O2, then the newly oxygenated blood returns to the heart. The other division is the systemic circuitry which transports the freshly oxygenated blood to the body tissues and picks up deoxygenated blood along its way back to the heart. Suggested Reading Activity Seeley, Rod R. et. al. 2006. Anatomy and Physiology. 7th Edition. New York, New York: McGraw-Hill Companies, Inc. Chapter 19, pp. 652-674 Chapter 20, pp. 680-717 Chapter 23, pp. 829-869 Martini, Frederic H. 2002. Fundamentals of Anatomy and Physiology. 6th Edition. New York, New York: McGraw-Hill Companies, Inc. Ventilation During inspiration, air flows from the environment into the trachea, bronchi, bronchioles, and alveoli. During expiration, alveolar gas travels the same route in reverse. Physical factors that govern air flow in and out of the lungs are collectively referred to as the mechanics of ventilation and include air pressure variances, resistance to air flow, and lung compliance (Porth, 2008). Air Pressure Variances

Air flows from a region of higher pressure to a region of lower pressure. During inspiration, movement of the diaphragm and other muscles of respiration enlarges the thoracic cavity and thereby lowers the pressure inside the thorax to a level below that of atmospheric pressure. As a result, air is drawn through the trachea and bronchi into the alveoli (Porth, 2008). During normal expiration, the diaphragm relaxes and the lungs recoil, resulting in a decrease in the size of the thoracic cavity. The alveolar pressure then exceeds atmospheric pressure, and air flows from the lungs into the atmosphere (Porth, 2008).

Airway Resistance Resistance is determined chiefly by the radius or size of the airway through which the air is flowing. Any process that changes the bronchial diameter or width affects airway resistance and alters the rate of air flow for a given pressure gradient during respiration. With increased resistance, greater-than-normal respiratory effort is required by the patient to achieve normal levels of ventilation (Porth, 2008). Compliance The pressure gradient between the thoracic cavity and the atmosphere causes air to flow in and out of the lungs. When pressure changes are applied in the normal lung, there is a proportional change in the lung volume. A measure of the elasticity, expandability, and distensibility of the lungs and thoracic structures is called compliance. Factors that determine lung compliance are the surface tension of the alveoli (normally low with the presence of surfactant) and the connective tissue (i.e., collagen and elastin) of the lungs (Porth, 2008). Exchange of Gases Between The Atmosphere and the Lungs Basic Properties of Gases The air we breathe is made up of a mixture of gases, mainly nitrogen and oxygen. These gases exert a combined pressure called the atmospheric pressure. The pressure at sea level, which is

defined as one atmosphere, is 760 mm of Hg (mm Hg), or 14.7 lbs/in 2 (PSI). When measuring respiratory pressures, atmospheric pressure is assigned a value of 0. A respiratory pressure of +15 mm Hg means that the pressure is 15 mm Hg above atmospheric pressure, and a respiratory pressure of 15 mm Hg is 15 mm Hg less than atmospheric pressure. Respiratory pressures often are expressed in centimeters of water (cm H2O) because of the small pressures involved (1 mm Hg = 1.35 cm H2O pressure) (Porth, 2008). The pressure exerted by a single gas in a mixture is called the partial pressure. The capital letter P followed by the chemical symbol of the gas (PO2) is used to denote its partial pressure. The law of partial pressures states that the total pressure of a mixture of gases, as in the atmosphere, is equal to the sum of the partial pressures of the different gases in the mixture. If the concentration of oxygen at 760 mm Hg (1 atmosphere) is 20%, its partial pressure is 152 mm Hg (760 0.20) (Porth, 2008). Warm air holds more moisture than cold air. This is the reason that precipitation in the form of rain or snow commonly occurs when the relative humidity is high and there is a sudden drop in atmospheric temperature. The air in the alveoli, which is 100% saturated at normal body temperature, has a water vapor pressure of 47 mm Hg. The water vapor pressure must be included in the sum of the total pressure of the gases in the alveoli (i.e., the total pressure of the other gases in the alveoli is 76047= 713 mm Hg). Air moves between the atmosphere and the lungs because of a pressure difference (Porth, 2008). VENTILATION AND THE MECHANICS OF BREATHING Ventilation is concerned with the movement of gases into and out of the lungs. There is nothing mystical about ventilation. It is purely a mechanical event that obeys the laws of physics as they relate to the behavior of gases. It relies on a system of open airways and the respiratory pressures created as the movements of the respiratory muscles change the size of the chest cage. The degree to which the lungs inflate and deflate depends on the respiratory pressures inflating the lung, compliance of the lungs, and airway resistance (Porth, 2008). Respiratory Pressures

The pressure inside the airways and alveoli of the lungs is called the intrapulmonary pressure or alveolar pressure. The gases in this area of the lungs are in communication with atmospheric pressure. When the glottis is open and air is not moving into or out of the lungs, as occurs just before inspiration or expiration, the intrapulmonary pressure is zero or equal to atmospheric pressure. The pressure in the pleural cavity is called the intrapleural pressure. The intrapleural pressure is always negative in relation to alveolar pressure, approximately 4 mm Hg between breaths when the glottis is open and the alveolar spaces are open to the atmosphere (Porth, 2008). The lungs and the chest wall have elastic properties, each pulling in the opposite direction. If removed from the chest, the lungs would contract to a smaller size, and the chest wall, if freed from the lungs, would expand. The opposing forces of the chest wall and lungs create a pull against the visceral and parietal layers of the pleura, causing the pressure in the pleural cavity to become negative. During inspiration, the elastic recoil of the lungs increases, causing intrapleural pressure to become more negative than during expiration. Without the negative intrapleural pressure holding the lungs against the chest wall, their elastic recoil properties would cause them to collapse. Although intrapleural pressure is negative in relation to alveolar pressure, it may become positive in relation to atmospheric pressure (e.g., during forced expiration and coughing) (Porth, 2008). The intrathoracic pressure is the pressure in the thoracic cavity. It is essentially equal to intrapleural pressure and is the pressure to which the lungs, heart, and great vessels are exposed. Forced expiration against a closed glottis compresses the air in the thoracic cavity and produces marked increases in intrathoracic pressure and intrapleural pressure (Porth, 2008). The Chest Cage and Respiratory Muscles The lungs and major airways share the chest cavity with the heart, great vessels, and esophagus. The chest cavity is a closed compartment bounded on the top by the neck muscles and at the bottom by the diaphragm. The outer walls of the chest cavity are formed by 12 pairs of ribs, the sternum, the thoracic vertebrae, and the intercostal muscles that lie between the ribs. Mechanically, the act of breathing depends on the fact that the chest cavity is a closed compartment whose only opening to the external atmosphere is through the trachea (Porth, 2008).

During normal levels of inspiration, the diaphragm moves approximately 1 cm, but this can be increased to 10 cm on forced inspiration. The diaphragm is innervated by the phrenic nerve roots, which arise from the cervical level of the spinal cord, mainly from C4 but also from C3 and C5. Paralysis of one side of the diaphragm causes the chest to move up on that side rather than down during inspiration because of the negative pressure in the chest. This is called paradoxical movement (Porth, 2008). The external intercostal muscles, which also aid in inspiration, connect to the adjacent ribs and slope downward and forward. When they contract, they raise the ribs and rotate them slightly so that the sternum is pushed forward; this enlarges the chest from side to side and from front to back. The intercostal muscles receive their innervation from nerves that exit the central nervous system at the thoracic level of the spinal cord. Paralysis of these muscles usually does not have a serious effect on respiration because of the effectiveness of the diaphragm. The accessory muscles of inspiration include the scalene muscles and the sternocleidomastoid muscles. The scalene muscles elevate the first two ribs, and the sternocleidomastoid muscles raise the sternum to increase the size of the chest cavity. These muscles contribute little to quiet breathing but contract vigorously during exercise (Porth, 2008). Lung Compliance Lung compliance refers to the ease with which the lungs can be inflated. It is determined by the elastin and collagen fibers of the lung, its water content, and surface tension. Compliance can be appreciated by comparing the ease of blowing up a noncompliant new balloon that is stiff and resistant with a compliant one that has been previously blown up and stretched. Specifically, lung compliance (C) describes the change in lung volume (V) that can be accomplished with a given change in respiratory pressure (P) (Porth, 2008). C= V / P The normal compliance of both lungs in the average adult is approximately 200 mL/cm H2O. This means that every time the transpulmonary pressure increases by 1 cm/ H2O, the lung volume expands by 200 mL. It would take more pressure to move the same amount of air into a noncompliant lung (Porth, 2008). Changes in Elastin and Collagen Composition of Lung

Tissue. Lung tissue is made up of elastin and collagen fibers. The elastin fibers are easily stretched and increase the ease of lung inflation, whereas the collagen fibers resist stretching and make lung inflation more difficult. In lung diseases such as interstitial lung disease and pulmonary fibrosis, the lungs become stiff and noncompliant as the elastin fibers are replaced with scar tissue. Pulmonary congestion and edema produce a reversible decrease in pulmonary compliance. Elastic recoil describes the ability of the elastic components of the lung to recoil to their original position after having been stretched. Overstretching the airways, as occurs with emphysema, causes the elastic components of the lung to lose their recoil, making the lung easier to inflate but more difficult to deflate because of its inability to recoil (Porth, 2008).

Surface Tension. An important factor in lung compliance is the surface tension in the alveoli. The alveoli are lined with a thin film of liquid, and it is at the interface between this liquid film and the alveolar air that surface tension develops. This is because the forces that hold the liquid film molecules together are stronger than those that hold the air molecules in the alveoli together. As an example, it is surface tension that holds the water molecules in a raindrop together. In the alveoli, excess surface tension causes the liquid film to contract, making lung inflation more difficult. The pressure in the alveoli (which are modeled as spheres with open airways projecting from them) can be predicted using Laplaces law (pressure = 2 surface tension/ radius). If the surface tension were equal throughout the lungs, the alveoli with the smallest radii would have the greatest pressure, and this would cause them to empty into the larger alveoli. The reason this does not occur is because of special surface tensionlowering molecules, called surfactant, that line the inner surface of the alveoli (Porth, 2008).

Law of Laplace: (P = 2 T/r, P = pressure, T = tension, r = radius). The effect of the radius on the pressure and movement of

gases in the alveolar structures is depicted. Air moves from P1 with a small radius and higher pressure to P2 with its larger radius and lower pressure. Surfactant is a complex mixture of lipoproteins (largely phospholipids) and small amounts of carbohydrates that is synthesized in the type II alveolar cells. The surfactant molecule has two ends: a hydrophobic (water-insoluble) tail and a hydrophilic (water-soluble) head. The hydrophilic head of the surfactant molecule attaches to the liquid molecules and the hydrophobic tail to the gas molecules, interrupting the intermolecular forces that are responsible for creating the surface tension (Porth, 2008). Surfactant exerts four important effects on lung inflation: it lowers the surface tension; it increases lung compliance and ease of inflation; it provides for stability and more even inflation of the alveoli; and it assists in preventing pulmonary edema by keeping the alveoli dry. Without surfactant, lung inflation would be extremely difficult, requiring intrapleural pressures of 20 to 30 mm Hg, compared with the pressures of 3 to 5 mm Hg that normally are needed (Porth, 2008). The surfactant molecules are more densely packed in the small alveoli than in larger alveoli, where the density of the molecules is less. Therefore, surfactant reduces the surface tension more effectively in the small alveoli, which have the greatest tendency to collapse, providing for stability and more even distribution of ventilation. Surfactant also helps to keep the alveoli dry and prevent pulmonary edema. This is because water is pulled out of the pulmonary capillaries into the alveoli when increased surface tension causes the alveoli to contract. The type II alveolar cells that produce surfactant do not begin to mature until the 26th to 28th week of gestation; consequently, many premature infants have difficulty producing sufficient amounts of surfactant. This can lead to alveolar collapse and severe respiratory distress. This condition, called infant respiratory distress syndrome, is the single most common cause of respiratory disease in premature infants. Surfactant dysfunction also is possible in the adult. This usually occurs as the result of severe injury or infection and can contribute to the development of a condition called adult respiratory distress syndrome (Porth, 2008). Airway Resistance

The volume of air that moves into and out of the air exchange portion of the lungs is directly related to the pressure difference between the lungs and the atmosphere and inversely related to the resistance that the air encounters as it moves through the airways. Airway resistance is the ratio of the pressure driving inspiration or expiration to airflow (Porth, 2008). The French physician Jean Leonard Marie Poiseuille first described the pressure flow characteristics of laminar flow in a straight circular tube, a correlation that has become known as Poiseuilles Law. According to Poiseuilles law, the resistance to flow is inversely related to the fourth power of the radius (R=1/r 4). If the radius is reduced by one half, the resistance increases 16-fold (2 x 2 x 2 x 2=16) (Porth, 2008). Airway resistance normally is so small that only small changes in pressure are needed to move large volumes of air into the lungs. For example, the average pressure change that is needed to move a normal breath of 500 mL of air into the lungs is approximately 1-2 cm H2O. Because the resistance of the airways is inversely proportional to the fourth power of the radius, small changes in airway caliber, such as those caused by pulmonary secretions or bronchospasm, can produce a marked increase in airway resistance. For persons with these conditions to maintain the same rate of airflow as before the onset of increased airway resistance, an increase in driving pressure (i.e., respiratory effort) is needed (Porth, 2008). Airway resistance is greatly affected by lung volumes, being less during inspiration than during expiration. This is because elastic-type fibers connect the outside of the airways to the surrounding lung tissues. As a result, these airways are pulled open as the lungs expand during inspiration, and they become narrower as the lungs deflate during expiration. This is one of the reasons that persons with conditions that increase airway resistance, such as bronchial asthma, usually have less difficulty during inspiration than during expiration (Porth, 2008). Laminar Versus Turbulent Flow. Airflow can be laminar or turbulent, depending on the velocity and pattern of flow. Laminar, or streamlined, airflow occurs at low flow rates in which the airstream is parallel to the sides of the airway. With laminar flow, the air at the periphery must overcome the resistance to flow, and as a result, the air in the center of the airway moves faster.

Turbulent flow is disorganized flow in which the molecules of the gas move laterally, collide with one another, and change their velocities. Whether turbulence develops depends on the radius of the airways, the interaction of the gas molecules, and the velocity of airflow. It is most likely to occur when the radius of the airways is large and the velocity of flow is high. Turbulent flow occurs regularly in the trachea. Turbulence of airflow accounts for the respiratory sounds that are heard during chest auscultation (i.e., listening to chest sounds using a stethoscope). In the bronchial tree with its many branches, laminar airflow probably occurs only in the very small airways, where the velocity of flow is low. Because the small airways contribute little resistance to airflow, they constitute a silent zone. In small airway disease (e.g., chronic obstructive pulmonary disease), it is probable that considerable abnormalities are present before the usual measurements of airway resistance can detect them (Porth, 2008). Airway Compression. Airflow through the collapsible airways in the lungs depends on the distending airway (intrapulmonary) pressures that hold the airways open and the external (intrapleural or intrathoracic) pressures that surround and compress the airways. The difference between these two pressures (intrathoracic pressure minus airway pressure) is called the transpulmonary pressure. For airflow to occur, the distending pressure inside the airways must be greater than the compressing pressure outside the airways. During forced expiration, the transpulmonary pressure is decreased because of a disproportionate increase in the intrathoracic pressure compared with airway pressure. The resistance that air encounters as it moves out of the lungs causes a further drop in airway pressure. If this drop in airway pressure is sufficiently great, the surrounding intrathoracic pressure will compress the collapsible airways (i.e., those that lack cartilaginous support), causing airflow to be interrupted and air to be trapped in the terminal airways. Although this type of airway compression usually is seen only during forced expiration in persons with normal respiratory function, it may occur during normal breathing in persons with lung diseases (Porth, 2008). Question No. 5 In clients with Emphysema, how does pursed-lip breathing increase airway pressure and improve expiratory flow rates? Explain the mechanisms involved.

LUNG VOLUMES Lung volumes, or the amount of air exchanged during ventilation, can be subdivided into three components: the tidal volume, the inspiratory reserve volume, and the expiratory reserve volume. The tidal volume (TV), usually about 500 mL, is the amount of air that moves into and out of the lungs during a normal breath. The maximum amount of air that can be inspired in excess of the normal TV is called the inspiratory reserve volume (IRV), and the maximum amount that can be exhaled in excess of the normal TV is the expiratory reserve volume (ERV) (Porth, 2008). Approximately 1200 mL of air always remains in the lungs after forced expiration; this air is the residual volume (RV). The RV increases with age because there is more trapping of air in the lungs at the end of expiration. These volumes can be measured using an instrument called a spirometer. Lung capacities include two or more lung volumes. The vital capacity equals the IRV plus the TV plus the ERV and is the amount of air that can be exhaled from the point of maximal inspiration. The inspiratory capacity equals the TV plus the IRV. It is the amount of air a person can breathe in beginning at the normal expiratory level and distending the lungs to the maximal amount. The functional residual capacity is the sum of the RV and ERV; it is the volume of air that remains in the lungs at the end of normal expiration. The total lung capacity is the sum of all the volumes in the lungs. The RV cannot be measured with the spirometer because this air cannot be expressed from the lungs. It is measured by indirect methods, such as the helium dilution method, the nitrogen washout techniques, or body plethysmography (Porth, 2008). Pulmonary Function Studies The previously described lung volumes and capacities are anatomic or static measures, determined by lung volumes and measured without relation to time. The spirometer also is used to measure dynamic lung function (i.e., ventilation with respect to time); these tests often are used in assessing pulmonary function. Pulmonary function measures include maximum voluntary ventilation, forced vital capacity, forced expiratory volumes and flow rates, and forced inspiratory flow rates. Pulmonary function is measured for various clinical purposes, including diagnosis of respiratory disease,

preoperative surgical and anesthetic risk evaluation, and symptom and disability evaluation for legal or insurance purposes. The tests also are used for evaluating dyspnea, cough, wheezing, and abnormal radiologic or laboratory findings (Porth, 2008). The maximum voluntary ventilation measures the volume of air that a person can move into and out of the lungs during maximum effort lasting for 12-15 secs. This measurement usually is converted to liters per minute. The forced expiratory vital capacity (FVC) involves full inspiration to total lung capacity followed by forceful maximal expiration. Obstruction of airways produces an FVC that is lower than that observed with more slowly performed vital capacity measurements. The forced expiratory volume (FEV) is the expiratory volume achieved in a given time period. The FEV1.0 is the forced expiratory volume that can be exhaled in 1 second. The FEV1.0 frequently is expressed as a percentage of the FVC. The FEV1.0 and FVC are used in the diagnosis of obstructive lung disorders. The forced inspiratory vital flow (FIF) measures the respiratory response during rapid maximal inspiration. Calculation of airflow during the middle half of inspiration (FIF25%75%) relative to the forced midexpiratory flow rate (FEF25%75%) is used as a measure of respiratory muscle dysfunction because inspiratory flow depends more on effort than does expiration (Porth, 2008). Suggested Reading Activity Porth, Carol Mattson. 2008. Pathophysiology: Concepts of Altered Health States. 7th Edition. Philadelphia, USA: Lippincott Williams and Wilkins Bullock, Barbara L. and Henze, Reet L. 2000. Focus on Pathophysiology. Philadelphia, USA: Lippincott Williams & Wilkins.

EFFICIENCY AND THE WORK OF BREATHING The minute volume, or total ventilation, is the amount of air that is exchanged in 1 minute. It is determined by the metabolic needs of the body. The minute volume is equal to the TV multiplied by the respiratory rate, which is normally about 6000 mL (500 mL TV x respiratory rate of 12 breaths/minute) during normal activity. The efficiency of breathing is determined by matching the TV and respiratory rate in a manner that provides an optimal minute volume while minimizing the work of breathing. The work of breathing is

determined by the amount of effort required to move air through the conducting airways and by the ease of lung expansion, or compliance. Expansion of the lungs is difficult for persons with stiff and noncompliant lungs; they usually find it easier to breathe if they keep their TV low and breathe at a more rapid rate (e.g., 300 x 20 = 6000 mL) to achieve their minute volume and meet their O2 needs. In contrast, persons with obstructive airway disease usually find it less difficult to inflate their lungs but expend more energy in moving air through the airways. As a result, these persons take deeper breaths and breathe at a slower rate (e.g., 600 x 10 = 6000 mL) to achieve their O2 needs (Porth, 2008) . Efficiency of Diffusion at the Respiratory Membrane Gas exchange at the respiratory membrane is efficient for the following five reasons: 1. The Differences in Partial Pressure Across the Respiratory Membrane Are Substantial. This fact is important, because the greater the difference in partial pressure, the faster is the rate of gas diffusion. Conversely, if PO2 in alveoli decreases, the rate of O2 diffusion into blood will drop. This is why many people feel light-headed at altitudes of 3000 m or more; the partial pressure of O2 in their alveoli has dropped low enough that the rate of O2 absorption is significantly reduced (Martini, 2002). 2. The Distances Involved in Gas Exchange Are Small. The fusion of capillary and alveolar basal laminae reduces the distance for gas exchange to an average of 0.5 um. Inflammation of the lung tissue of a buildup of fluid in alveoli increases the diffusion distance and impairs alveolar gas exchange (Martini, 2002). 3. The Gases Are Lipid Soluble. Both O2 and CO2 diffuse readily through the surfactant layer and the alveolar and endothelial cell membranes (Martini, 2002). 4. The Total Surface Area Is Large. The combined alveolar surface area at peak inhalation may approach 140 m2 (1,506 ft2) (Martini, 2002). 5. Blood Flow and Airflow Are Coordinated. This arrangement improves the efficiency of both pulmonary ventilation and pulmonary circulation. For example, blood flow is greatest around alveoli with the

highest PO2 values, where O2 uptake can proceed with maximum efficiency. If the normal blood flow is impaired or if the flow is interrupted, this coordination is lost and respiratory efficiency decreases (Martini, 2002).

Question No. 6 Which of the following factors stated above is affected in a case of Pulmonary Embolism? Explain how is it affected by the disorder?

EXCHANGE AND TRANSPORT OF GASES The primary functions of the lungs are oxygenation of the blood and removal of CO2. Pulmonary gas exchange is conventionally divided into three processes: ventilation, or the flow of gases into and out of the alveoli of the lungs; perfusion, or flow of blood in the adjacent pulmonary capillaries; and diffusion, or transfer of gases between the alveoli and the pulmonary capillaries. The efficiency of gas exchange requires that alveolar ventilation occur adjacent to perfused pulmonary capillaries (Porth, 2008). Ventilation Ventilation refers to the exchange of gases in the respiratory system. There are two types of ventilation: pulmonary and alveolar. Pulmonary ventilation refers to the total exchange of gases between the atmosphere and the lungs. Alveolar ventilation is the exchange of gases within the gas exchange portion of the lungs. Ventilation requires a system of open airways and a pressure difference that moves air into and out of the lungs. It is affected by body position and lung volume as well as by disease conditions that affect the heart and respiratory systems (Porth, 2008). Distribution of Ventilation The distribution of ventilation between the apex and base of the lung varies with body position and the weight of the lung and the effects of gravity on intrapleural pressure. Compliance reflects the change in volume that occurs with a change in pressure. It is less in fully expanded alveoli, which have difficulty accommodating more air,

and greater in alveoli that are less inflated. In the seated or standing position, gravity exerts a downward pull on the lung, causing intrapleural pressure at the apex of the lung to become more negative than that at the base of the lung. As a result, the alveoli at the apex of the lung are more fully expanded and less compliant than those at the base of the lung. The same holds true for lung compliance in the dependent portions of the lung in the supine or lateral position. In the supine position, ventilation in the lowermost (posterior) parts of the lung exceeds that in the uppermost (anterior) parts. In the lateral position (i.e., lying on the side), the dependent lung is better ventilated. The distribution of ventilation also is affected by lung volumes. During full inspiration in the seated or standing position, the airways are pulled open, and air moves into the more compliant portions of the lower lung. At low lung volumes, the opposite occurs. At functional residual capacity, the pleural pressure at the base of the lung exceeds airway pressure compressing the airways, so that ventilation is greatly reduced (Porth, 2008). Perfusion The primary functions of the pulmonary circulation are to perfuse or provide blood flow to the gas exchange portion of the lung and to facilitate gas exchange. The pulmonary circulation serves several important functions in addition to gas exchange. It filters all the blood that moves from the right to the left side of the circulation; it removes most of the thromboemboli that might form; and it serves as a reservoir of blood for the left side of the heart. The gas exchange function of the lungs requires a continuous flow of blood through the respiratory portion of the lungs (Porth, 2008). Deoxygenated blood enters the lung through the pulmonary artery, which has its origin in the right side of the heart and enters the lung at the hilus, along with the primary bronchus. The pulmonary arteries branch in a manner similar to that of the airways. The small pulmonary arteries accompany the bronchi as they move down the lobules and branch to supply the capillary network that surrounds the alveoli. The meshwork of capillaries in the respiratory portion of the lungs is so dense that the flow in these vessels often is described as being similar to a sheet of blood. The oxygenated capillary blood is collected in the small pulmonary veins of the lobules; it then moves to the larger veins to be collected in the four large pulmonary veins that empty into the left atrium (Porth, 2008).

Distribution of Blood Flow and Body Position As with ventilation, the distribution of pulmonary blood flow is affected by body position and gravity. In the upright position, the distance of the upper apices of the lung above the level of the heart may exceed the perfusion capabilities of the mean pulmonary arterial pressure (approximately 12 mm Hg); therefore, blood flow in the upper part of the lungs is less than that in the base or bottom part of the lungs (Porth, 2008). In the supine position, the lungs and the heart are at the same level, and blood flow to the apices and base of the lungs becomes more uniform. In this position, blood flow to the posterior or dependent portions (e.g., bottom of the lung when lying on the side) exceeds flow in the anterior or nondependent portions of the lungs (Porth, 2008). Hypoxia-induced Vasoconstriction The blood vessels in the pulmonary circulation undergo marked vasoconstriction when they are exposed to hypoxia. The precise mechanism for this response is unclear. When alveolar oxygen levels drop below 60 mm Hg, marked vasoconstriction may occur, and at very low oxygen levels, the local flow may be almost abolished. In regional hypoxia, as occurs with atelectasis, vasoconstriction is localized to a specific region of the lung. Vasoconstriction has the effect of directing blood flow away from the hypoxic regions of the lungs. When alveolar hypoxia no longer exists, blood flow is restored. Generalized hypoxia causes vasoconstriction throughout the lung. Generalized vasoconstriction occurs when the partial pressure of oxygen is decreased at high altitudes, or it can occur in persons with chronic hypoxia due to lung disease. Prolonged hypoxia can lead to pulmonary hypertension and increased workload on the right heart. A low blood pH also produces vasoconstriction, especially when alveolar hypoxia is present (e.g., during circulatory shock) (Porth, 2008). Diffusion There are two types of air movement in the lung: bulk flow and diffusion. Bulk flow occurs in the conducting airways and is controlled by pressure differences between the mouth and the airways in the lung. Diffusion refers to the movement of gases in the alveoli and across the alveolar capillary membrane. Gas diffusion in the lung can be described by Ficks Law (Porth, 2008).

Ficks law states that the volume of a gas (V gas) diffusing across the membrane per unit time is directly proportional to the partial pressure difference of the gas (P1 P2), the surface area (SA) of the membrane, and the diffusion coefficient (D) and is inversely proportional to the thickness (T) of the membrane (Porth, 2008): Vgas = (P1-P2) x SA X D T Several factors influence diffusion of gases in the lung. The administration of high concentrations of oxygen increases the difference in partial pressure between the two sides of the membrane and increases the diffusion of the gas. Diseases that destroy lung tissue (i.e., surface area for diffusion) or increase the thickness of the alveolar-capillary membrane adversely influence the diffusing capacity of the lungs. The removal of one lung, for example, reduces the diffusing capacity by one half. The thickness of the alveolar-capillary membrane and the distance for diffusion are increased in persons with pulmonary edema or pneumonia. The characteristics of the gas and its molecular weight and solubility constitute the diffusion coefficient and determine how rapidly the gas diffuses through the respiratory membranes. For example, CO2 diffuses 20 times more rapidly than O2 because of its greater solubility in the respiratory membranes (Porth, 2008).

Matching of Ventilation and Perfusion The gas exchange properties of the lung depend on matching ventilation and perfusion, ensuring that equal amounts of air and blood are entering the respiratory portion of the lungs. Two factors may interfere with the matching of ventilation and perfusion: dead air space and shunt (Porth, 2008). Dead Air Space Dead space refers to the volume of air that must be moved with each breath but does not participate in gas exchange. The movement of air through dead space contributes to the work of breathing but not to gas exchange. There are two types of dead air space: that contained in the conducting airways, called the anatomic dead space, and that contained in the respiratory portion of the lung, called

the alveolar dead space. The volume of anatomic airway dead space is fixed at approximately 150-200 mL, depending on body size. It constitutes air contained in the nose, pharynx, trachea, and bronchi. The creation of a tracheostomy decreases anatomic dead space ventilation because air does not have to move through the nasal and oral airways. Alveolar dead space, normally about 5-10 mL, constitutes alveolar air that does not participate in gas exchange. When alveoli are ventilated but deprived of blood flow, they do not contribute to gas exchange and thereby constitute alveolar dead space. The physiologic dead space includes the anatomic dead space plus alveolar dead space. In persons with normal respiratory function, physiologic dead space is about the same as anatomic dead space. Only in lung disease does physiologic dead space increase. Alveolar ventilation is equal to the minute ventilation minus the physiologic dead space ventilation (Porth, 2008). Shunt Shunt refers to blood that moves from the right to the left side of the circulation without being oxygenated. As with dead air space, there are two types of shunts: physiologic and anatomic. In an anatomic shunt, blood moves from the venous to the arterial side of the circulation without moving through the lungs. In a physiologic shunt, there is mismatching of ventilation and perfusion with the lung, resulting in insufficient ventilation to provide the O2 needed to oxygenate the blood flowing through the alveolar capillaries. Physiologic shunting of blood usually results from destructive lung disease that impairs ventilation or from heart failure that interferes with movement of blood through sections of the lungs (Porth, 2008). Mismatching of Ventilation and Perfusion Both dead air space and shunt produce a mismatching of ventilation and perfusion. With shunt (depicted on the left), there is perfusion without ventilation, resulting in a low ventilationperfusion ratio. It occurs in conditions such as atelectasis in which there is airway obstruction. With dead air space (depicted on the right), there is ventilation without perfusion, resulting in a high ventilation perfusion ratio. It occurs with conditions such as pulmonary embolism, which impairs blood flow to a part of the lung. The arterial blood leaving the pulmonary circulation reflects mixing of blood from normally ventilated and perfused areas of the lung as well as areas that are not ventilated (dead air space) or perfused (shunt). Many of the conditions that cause mismatching of ventilation and perfusion

involve both dead air space and shunt. In chronic obstructive lung disease, for example, there may be impaired ventilation in one area of the lung and impaired perfusion in another area (Porth, 2008).

GAS TRANSPORT Although the lungs are responsible for the exchange of gases, it is the blood that transports these gases between the lungs and body tissues. The blood carries O2 and CO2 in the dissolved state and in combination with hemoglobin. CO2 also is converted to bicarbonate and transported in that form. The amount of a gas that can dissolve in plasma is determined by two factors: the solubility of the gas in the plasma and the partial pressure of the gas in the alveoli (Porth, 2008). In the clinical setting, blood gas measurements are used to determine the level of the partial pressure of oxygen (PO2) and carbon dioxide (PCO2) in the blood. Arterial blood commonly is used for measuring blood gases. Venous blood is not used because venous levels of O2 and CO2 reflect the metabolic demands of the tissues rather than the gas exchange function of the lungs. The PO2 of arterial blood normally is above 80 mm Hg, and the PCO2 is in the range of 3545 mm Hg. Normally, the arterial blood gases are the same or nearly the same as the partial pressure of the gases in the alveoli. The arterial PO2 often is written PaO2, and the alveolar PO2 as PAO2, with the same types of designations being used for PCO2. This text uses PO2 and PCO2 to designate both arterial and alveolar levels of the gases (Porth, 2008). Oxygen Transport Oxygen is transported in two forms: in chemical combination with hemoglobin and in the dissolved state. The hemoglobin (hgb) in red blood cells serves as a transport vehicle for O2. It binds O2 in the pulmonary capillaries and releases it in the tissue capillaries. As O2 moves into or out of the RBCs, it dissolves in the plasma. It is the dissolved form of O2 that leaves the capillary, crosses cell membranes, and participates in cell metabolism (Porth, 2008). Dissolved Oxygen. The partial pressure of oxygen (PO2) represents the level of dissolved O2 in plasma. The amount of gas that can be dissolved in a

liquid depends on the solubility of the gas and its pressure. The solubility of O2 in plasma is fixed and very small. For every 1 mm Hg of PO2 present in the alveoli, 0.003 mL of O2 becomes dissolved in 100 mL of plasma. This means that at a normal alveolar PO2 of 100 mm Hg, the blood carries only 0.3 mL of dissolved O2 in each 100 mL of plasma. This amount (approximately 1%) is very small compared with the amount that can be carried in an equal amount of blood when O2 is attached to hgb. Hemoglobin Transport. Hemoglobin is a highly efficient carrier of O2, and approximately 98% to 99% of the O2 used by body tissues is carried in this manner. Hemoglobin with bound O2 is called oxyhemoglobin, and when oxygen is removed, it is called deoxygenated or reduced hemoglobin. Each gram of hemoglobin carries approximately 1.34 mL of oxygen when it is fully saturated. This means that a person with a hemoglobin (Hgb) of 14 g/100 mL carries 18.8 mL of O2 per 100 mL of blood. In the lungs, O2 moves across the alveolar-capillary membrane, through the plasma, and into the RBC, where it forms a loose and reversible bond with the Hgb molecule. In normal lungs, this process is rapid, so that even with a fast heart rate, the Hgb is almost completely saturated with O2 during the short time it spends in the pulmonary capillaries (Porth, 2008). The oxygenated hgb is transported in the arterial blood to the peripheral capillaries, where the O2 is released and made available to the tissues for use in cell metabolism. As the O2 moves out of the capillaries in response to the needs of the tissues, the hgb saturation, which usually is approximately 95%-97% as the blood leaves the left side of the heart, drops to approximately 75% as the mixed venous blood returns to the right side of the heart (Porth, 2008). Binding Affinity of Hemoglobin for Oxygen. Oxygen that remains bound to hgb cannot participate in tissue metabolism. The efficiency of the hgb transport system depends on the ability of the hgb molecule to bind O 2 in the lungs and release it as it is needed in the tissues. The affinity of hgb refers to its capacity to bind O2. Hgb binds O2 more readily when its affinity is increased and releases it more readily when its affinity is decreased (Porth, 2008). Oxygen binds cooperatively with the heme groups on the hgb molecule. After the first molecule of O2 binds to hgb, the molecule

undergoes a change in shape. As a result, the second and third molecules bind more readily, and binding of the fourth molecule is even easier. In a like manner, the unloading of the first O2 molecule enhances the unloading of the next molecule, and so on. Thus, the affinity of hgb for O2 changes with O2 saturation. Hemoglobins affinity for O2 is also influenced by pH, CO2 concentration, and temperature. Hemoglobin binds O2 more strongly under conditions of increased pH (alkalosis), decreased CO2 concentration, and decreased body temperature and releases it more readily under conditions of decreased pH (acidosis), increased CO2 concentration, and fever. (Porth, 2008). OxygenHemoglobin Dissociation Curve. The relation between the O2 carried in combination with hemoglobin and the PO2 of the blood is described by the oxygen hemoglobin dissociation curve. The x axis of the graph depicts the PO2 or dissolved oxygen; the left y axis, hemoglobin saturation; and the right y axis, the oxygen content. The PO2 reflects the partial pressure of the gas in the lung (i.e., the PO2 is approximately 100 mm Hg when room air is being breathed, but can rise to 200 mm Hg or higher whenoxygen enriched air is breathed). The hgb saturation reflects the amount of O2 that is carried by the hgb (Porth, 2008). The S-shaped O2-dissociation curve has a flat top portion representing binding of O2 by the hgb in the lungs and a steep portion representing its release into the tissue capillaries. The S shape of the curve reflects the effect that O2 saturation has on the conformation of the hgb molecule and its affinity for O2. At approximately 100 mm Hg PO2, a plateau occurs, at which point the hgb is approximately 98% saturated. Increasing the alveolar PO2 above this level does not increase the hgb saturation (Porth, 2008). Even at high altitudes, when the partial pressure of O2 is considerably decreased, the hemoglobin remains relatively well saturated. At 60 mm Hg PO2, for example, the hgb is still approximately 89% saturated. The steep portion of the dissociation curvebetween 60 and 40 mm Hgrepresents the removal of O2 from the hgb as it moves through the tissue capillaries. This portion of the curve reflects the fact that there is considerable transfer of O2 from hgb to the tissues with only a small drop in PO2, thereby ensuring a gradient for O2 to move into body cells (Porth, 2008).

In order to function as a buffer system, the affinity of hgb for O2 must change with the metabolic needs of the tissues. This change is represented by a shift to the right or left in the dissociation curve. A shift to the right indicates that the tissue PCO2 is greater for any given level of hgb saturation and represents reduced affinity of the hemoglobin for O2 at any given PCO2. It usually is caused by conditions such as fever or acidosis or by an increase in PCO2, which reflects increased tissue metabolism. High altitude and conditions such as pulmonary insufficiency, heart failure, and severe anemia also cause the O2 dissociation curve to shift to the right (Porth, 2008). A shift to the left in the O2 dissociation curve represents an increased affinity of hemoglobin for O2 and occurs in situations associated with a decrease in tissue metabolism, such as alkalosis, decreased body temperature, and decreased PCO2 levels. The degree of shift can be determined by the P50, or the partial pressure of O 2 that is needed to achieve a 50% saturation of hgb (Porth, 2008). Carbon Dioxide Transport Carbon dioxide is transported in the blood in three forms: as dissolved carbon dioxide (10%), attached to hemoglobin (30%), and as bicarbonate (60%). Acid-base balance is influenced by the amount of dissolved carbon dioxide and the bicarbonate level in the blood (Porth, 2008). As CO2 is formed during the metabolic process, it diffuses out of cells into the tissue spaces and then into the capillaries. The amount of dissolved CO2 that can be carried in plasma is determined by the partial pressure of the gas and its solubility coefficient (0.03 mL/100 mL/1 mm Hg PCO2). Carbon dioxide is 20 times more soluble in plasma than oxygen. Thus, the dissolved state plays a greater role in transport of CO2 compared with O2 (Porth, 2008). Most of the carbon dioxide diffuses into the red blood cells, where it either forms carbonic acid or combines with hgb. Carbonic acid (H2CO3) is formed when CO2 combines with H2O (CO2 + H2O = H+ + HCO3-). The process is catalyzed by an enzyme called carbonic anhydrase, which is present in large quantities in RBCs. Carbonic anhydrase increases the rate of the reaction between CO2 and water approximately 5000- fold. Carbonic acid readily ionizes to form bicarbonate (HCO3) and hydrogen (H+) ions. The hydrogen ions combine with the hgb, which is a powerful acid-base buffer, and the

bicarbonate ion diffuses into plasma in exchange for a chloride ion. This exchange is made possible by a special bicarbonate-chloride carrier protein in the RBC membrane. As a result of the bicarbonatechloride shift, the chloride and water content of the RBC is greater in venous blood than in arterial blood (Porth, 2008). Each 100 mL of blood leaving the alveolar capillaries carries away roughly 20 mL of O2. Of this amount, only about 0.3 mL (1.5%) consists of oxygen molecules in solution. The rest of the O2 molecules are bound to hemoglobin moleculesspecifically, to the iron ions in the center of heme units. Recall that the hgb molecule consists of four globular protein subunits, each containing a heme unit. Thus, each hgb molecule can bind four molecules of O 2, forming oxyhemoglobin. This is a reversible reaction that can be summarized as: Hb + O2

HbO2

Hemoglobin and PO2 An oxygen-hemoglobin saturation curve, or oxygenhemoglobin dissociation curve, is a graph that relates the saturation of haemoglobin to the partial pressure of O2. The binding and dissociation of O2 to Hgb is a typical reversible reaction. At equilibrium, O2 molecules bind to heme at the same rate that other O2 molecules are being released. If you increase the PO2, you shift the reaction to the right, and more O2 gets bound to hemoglobin, If you decrease the PO2, the reaction shifts to the left, and more O2 is released by Hgb. The graph of this relationship is a curve rather than a straight line, because the shape of the Hgb molecule changes slightly each time it binds an O2 molecule, and this change affects its ability to bind another O2 molecule. In other words, the attachment of the first O2 molecule makes it easier to bind the second; binding of the second promotes binding of the third; and binding of the third enhances binding of the fourth, and so on.

Question No. 7 In real life, entire families are killed each winter by leaky furnaces or space heaters. Murder of suicide victims who died in their cars inside a closed garage are popular characters for mystery writers. What is the cause of death? Explain the pathogenesis of this situation.

Hemoglobin and pH In addition to consuming O2, active tissues generate acids that lower the pH of the interstitial fluid. When the pH drops, the shape of Hgb molecules changes. As a result of this change, the molecules release their O2 reserves more readily, so the slope of the hemoglobin saturation curve changes. In other words, the saturation declines. Thus, a tissue PO2 of 40 mm Hg, hgb molecules release 15% more O2 at a pH of 7.2 than they do at a pH of 7.4. This effect of pH on the Hgb saturation curve is called the Bohr effect. CO2 is the primary compound responsible for the Bohr effect. When CO2, diffuses into the blood, it rapidly diffuses into the RBCs. There, an enzyme called carbonic anhydrase catalyzes the reaction of CO2 with H2O molecules: CO2 + H2O
carbonic

anhydrase

H2CO3

H+

+ HCO3-

The product of this enzymatic reaction, H2CO3-, is called carbonic acid, because it dissociates into a hydrogen ion (H+) and a bicarbonate ion (HCO3-). The rate of H2CO3- formation depends on the amount of CO2 in solution, which, as we noted earlier depends on the PCO2. When the PCO2 rises, the reaction proceeds from left to the right and the rate of H2CO3- formation accelerates. The H+ ions that are generated diffuse out of the RBCs, and the pH of the plasma drops. When the PCO2 declines, the reaction proceeds from the right to left; H+ ions then diffuse into the RBCs, so the pH of the plasma rises. Hemoglobin and Temperature Changes in the temperature also affect the slope of the Hgb saturation curve. As the temperature rises, Hgb releases more O2; as the temperature declines, Hgb holds O2 more tightly. Temperature effects are significant only in active tissues in which large amounts of heat are being generated. Hemoglobin and BPG

RBCs produce adenosine triphosphate (ATP) only by glycolysis, in which lactic acid is formed. The metabolic pathways involved in glycolysis in an RBC also generate the compound 2,3biphosphoglycerate, or BPG. Normal RBCs always contain BPG, which has a direct effect on O2 binding and release. For any partial pressure of O2, the higher the concentration of BPG, the more O 2 will be released by the Hb molecules. The concentration of BPG can be increased by thyroid hormones, growth hormone, epinephrine, androgens, and a high blood pH. These factors improve O2 delivery to the tissues because when BPG levels are elevated, Hb will release about 10% more O2 at a given PO2 than it would otherwise. Both BPG synthesis and the Bohr effect improve O2 delivery when the pH changes; BPG levels rise when the pH increases, and the Bohr effect appears when the pH decreases. The production of BPG decreases the RBCs age. Thus, the level of BPG can determine how long a blood bank can hold fresh whole blood. When BPG levels get too low, Hgb becomes firmly bound to the available O2. The blood is then useless for transfusions, because the RBCs will no longer release O2 to peripheral tissues, even at a disastrously low PO2. Question No. 8 Atmospheric pressure decreases with increasing altitude, and so do the partial pressures of the component gases, including O2. Despite the low alveolar PO2, millions of people live and work at high altitudes (mountainous regions). What are the important physiological adjustments that represent an excellent functional interplay between the respiratory, cardiovascular, and hematological systems? Instructional References: Bullock, Barbara L. and Henze, Reet L. 2000. Focus on Pathophysiology. Philadelphia, USA: Lippincott Williams & Wilkins. Martini, Frederic H. 2002. Fundamentals of Anatomy and Physiology. 6th Edition. New York, New York: McGraw-Hill Companies, Inc.

Porth, Carol Mattson. 2005. Pathophysiology Concepts of Altered Health States. Philadelphia, USA: Lippincott Williams and Wilkins Seeley, Rod R. et. al. 2006. Anatomy and Physiology. 7th Edition. New York, New York: McGraw-Hill Companies, Inc.

Unit II Concepts on Integration, Control, Perception, and Coordination Module 1 Neurology and Neural Control

Objectives After you have completed this module, you should be able to: 1. list two major anatomical divisions of the nervous system, and describe the characteristics of each division 2. describe the structure of a synapse, and explain the mechanism involved in synaptic activity. 3. describe the major types of neurotransmitters and neuromodulations, and discuss their effects on postsynaptic membranes.

Overview Humans interact with the environment through the nervous system, perceiving and responding to the stimuli that continually affect them. A complex system of connections and interconnections of nerve cells provides for perception, interim processing, and response. In addition, characteristics that endow humans with the ability to think, feel, reason, and remember evolve from the interacting neuronal networks of the brain (Bullock, 2000). The nervous system can be divided into two: the central and the peripheral nervous system. The fundamental structure in each division is the neuron.

The neuron and groups of neurons communicate within the environment of the central and peripheral nervous systems (Bullock, 2000). The nervous system, in coordination with the endocrine system, provides the means by which cell and tissue functions are integrated into a solitary, surviving organism. No part of the nervous system functions independently from other parts. In humans, who are thinking and feeling creatures, the effects of emotion can exert a strong influence on neural and hormonal control of body function. However, alterations in neural and endocrine function, particularly at the biochemical level, also can exert a strong influence on psychological behavior (Porth, 2008). In this unit, our focus will be drawn to mechanisms that coordinate the activities of the bodys organ systems. Each moment the world changes a little bit, and you must recognize and respond to many of those changes. In many cases, you respond immediately, yet even the simplest response requires some level of cooperation among systems. All of these functions are performed by the nervous system (Martini, 2002).

The Neural Tissue The nervous system includes all the neural tissue in the body. The basic functional units of the nervous system are individual cells called neurons. Supporting cells, or neuroglia separate and protect the neurons, provide a supportive framework for neural tissue, act as phagocytes, and help regulate the composition of the interstitial fluid. Neuroglia, also called glial cells, far outnumber neurons and account for roughly half the volume of the nervous system (Martini, 2002). Functionally, neurons are classified as sensory, motor, or association neurons. Sensory or afferent neurons deliver information from sensory receptors to the CNS. The cell bodies of sensory neurons are located in the peripheral sensory ganglia. Motor or efferent neurons carry instructions from the CNS to peripheral effectors in the peripheral tissue, organ, or organ system. Interneurons or association neurons are responsible for the distribution of sensory information and the coordination of motor activity. For example, one or more interneurons are situated between sensory neurons and motor neurons. The more complex the response to a given stimulus, the greater the number of interneurons involved. Interneurons are also involved with all higher functions, such as memory, planning, and learning (Martini, 2002). Neuron Function The neuron has the capability to generate and conduct electrochemical impulses. The cellular, cytoplasmic, and metabolic activities that maintain cell life are similar to those of other cells. Neurons are distinct with their structural synapses allowing them to conduct impulses from one to another (Martini, 2002).

Peripheral Axonal Degeneration and Regeneration An injured neuron may regenerate as long as its cell body remains relatively unharmed. However, serious damage to the cell body results in death of the entire neuron. A crushed or severed axon of a peripheral nerve fiber triggers certain cellular changes within a

few hours of injury. Changes in the axon distal (portion of axon cut off from the cell body) to the injury are referred to as Wallerian degeneration and are particularly dramatic because that portion has been severed from the metabolic control of the cell body. Initially, the distal portion swells due to influx of sodium and water. The terminal neurofilaments hypertrophy and Schwann cells proliferate. Mitochondrial damage results from uncontrolled calcium influx. The myelin sheath shrinks and retracts the nodes of Ranvier, where the remaining nerve fiber becomes exposed. The axon gradually disappears, and myelin disintegrates into fragments that are phagocytized by the Schwann cells and tissue macrophages (Adams, 1998). Cellular cytoplasm swells, displacing the nucleus toward the cell wall. In response, cellular metabolic activity, protein synthesis, and mitochondrial activity increase. Injured neurofibrils (delicate threads projecting into the axon from the cell body) attempt to grow back into their original placements and begin sprouting from the proximal portion of the injured axon within 7-14 days after injury. If the fibrils are successful in finding their way into the neurolemma, which generally remains intact, they generally grow at a rate of 1.5 mm per day. The remaining Schwann cells form a sheet of myelin around the restored neurofibril, and the nodes of Ranvier are reformed as the nerve regenerates. Early surgical repair of severed peripheral nerves facilitates nerve regeneration. Regeneration of injured nerves in the CNS is more difficult due to glial scarring, which frequently inhibits new fibrils from reaching their destinations (Bullock, 2000). Question/Activity No. 9 Pick two demyelinating diseases of the nervous system and elucidate its effect on sensation and motor control. Use a schematic diagram or flowchart.

THE NERVE CELL COMMUNICATION I. Neuronal Conduction Conduction of neuronal impulses depends on membrane potentials. Membrane potentials are generated because of disparity between cations and anions at the semipermeable and selectively permeable nerve cell membrane. Specific proteins at the cell

membrane allow the movement of ions and facilitate the nerve cell potential and impulse propagation (Bullock, 2000). Action Potential Unique to nerve, muscle, and gland cells is the change that can occur in a resting cell membrane potential when the cells are stimulated by electrical, chemical, or mechanical means. These stimuli can produce a sudden increase in cell membrane permeability to sodium, which results in a very brief, positive potential within the cell. The sequence of physiochemical events that results in an alteration in the resting potential lasts a few milliseconds and is called the action potential. Action potentials are propagated changes in the transmembrane potential that, once initiated, affect an entire excitable membrane (Bullock, 2000). Conduction Velocity in Nerve Fibers The velocity of nerve conduction is influenced by the myelinization and diameter of the axon. Myelin acts as an effective insulator and inhibits electrochemical conduction along the full segment of the membrane. This selectivity facilitates and increase in conduction velocity. As the current passes over the myelin and through the extracellular fluid, it enters the nodes of Ranvier at 1-mm to 2-mm intervals, where the membrane is permeable to the ions. Current propagation in myelinated fibers is known as salutatory conduction, implying a leaping of hopping phenomenon (Bullock, 2000). The wave of action potential leaks out and dissipates with distance in an unmyelinated fiber, whereas the myelinated fiber retains the action potential longer. The diameter of the fiber is an additional important factor contributing to nerve conduction velocity. Velocity is increased in large-diameter nerve fibers due to lower internal resistance and a quicker depolarization time (Bullock, 2000).

Suggested Reading Activity Porth, Carol Mattson. 2008. Pathophysiology: Concepts of Altered Health States. 7th Edition. Philadelphia, USA: Lippincott Williams and Wilkins

Bullock, Barbara L. and Henze, Reet L. 2000. Focus on Pathophysiology. Philadelphia, USA: Lippincott Williams & Wilkins. II. SYNAPTIC TRANSMISSION Neurons communicate with each other through structures known as synapses. Two types of synapses are found in the nervous system: electrical and chemical. Electrical synapses permit the passage of current-carrying ions through small openings called gap junctions that penetrate the cell junction of adjoining cells and allow current to travel in either direction. The gap junctions allow an action potential to pass directly and quickly from one neuron to another. They may link neurons having close functional relationships into circuits (Porth, 2008). The most common type of synapse is the chemical synapse. Chemical synapses involve special presynaptic and postsynaptic membrane structures, separated by a synaptic cleft. Presynaptic terminals secrete one and often several chemical messenger molecules (i.e., neurotransmitters or neuromodulators) into the synaptic cleft. Neurotransmitters diffuse into the synaptic cleft and unite with receptors on the postsynaptic membrane; this causes excitation or inhibition of the postsynaptic neuron by producing either hypopolarization or hyperpolarization of the postsynaptic membrane (Porth, 2008). Hypopolarization increases the excitability of the postsynaptic neuron by bringing the membrane potential closer to the threshold potential so that a smaller subsequent stimulus is needed to cause the neuron to fire (Porth, 2008). Hyperpolarization, on the other hand, brings the membrane potential further from the threshold and has the opposite effect. It has an inhibitory effect and decreases the likelihood that an action potential will be generated (Porth, 2008). In contrast to an electrical synapse, a chemical synapse serves as a rectifier, permitting only one-way communication. One-way conduction is a particularly important characteristic of chemical synapses. It is this specific transmission of signals to discrete and highly localized areas of the nervous system that allows it to perform the myriad functions of sensation, motor control, and memory.

Chemical synapses are the slowest component in progressive communication through a sequence of neurons, such as in a spinal reflex. In contrast to the conduction of electrical action potentials, each successive event at the chemical synapsetransmitter secretion, diffusion across the synaptic cleft, interaction with postsynaptic receptors, and generation of a subsequent action potential in the postsynaptic neuronconsumes time. On the average, conduction across a chemical synapse requires approximately 0.3 milliseconds. A neurons cell body and dendrites are covered by thousands of synapses, any or many of which can be active at any moment. Because of the interaction of this rich synaptic input, each neuron resembles a little integrator, in which circuits of many neurons interact with one another (Porth, 2008). VENTRICULAR SYSTEM AND CEREBROSPINAL FLUID The ventricular system is a series of CSF-filled cavities in the brain. Cerebrospinal fluid provides a supporting and protective fluid in which the brain and spinal cord float. CSF helps maintain a constant ionic environment that serves as a medium for diffusion of nutrients, electrolytes, and metabolic end products into the extracellular fluid surrounding CNS neurons and glia. Filling the ventricles, the CSF supports the mass of the brain. Because it fills the subarachnoid space surrounding the CNS, a physical force delivered to either the skull or spine is to some extent diffused and cushioned (Porth, 2008). The lining of the ventricles and central canal of the spinal cord is called the ependyma. There is a tremendous expansion of the ependyma in the roof of the lateral, 3rd, and 4th ventricles. The CSF is produced by tiny reddish masses of specialized capillaries from the pia mater, called the choroid plexus, that project into the ventricles. CSF is an ultrafiltrate of blood plasma, composed of 99% H2O with other constituents, making it close to the composition of the brain extracellular fluid. Humans secrete approximately 500 mL of CSF each day. However, only approximately 150 mL is in the ventricular system at any one time, meaning that the CSF is continuously being absorbed (Porth, 2008). The CSF produced in the ventricles must flow through the interventricular foramen, the third ventricle, the cerebral aqueduct, and the fourth ventricle to exit from the neural tube. Three openings,

or foramina, allow the CSF to pass into the subarachnoid space. Two of these, the foramina of Luschka, are located at the lateral corners of the fourth ventricle. The third, the medial foramen of Magendie, is in the midline at the caudal end of the fourth ventricle. Approximately 30% of the CSF passes down into the subarachnoid space that surrounds the spinal cord, mainly on its dorsal surface, and moves back up to the cranial cavity along its ventral surface (Porth, 2008). Reabsorption of CSF into the vascular system occurs along the sides of the superior sagittal sinus in the anterior and middle fossa. To reach this area, the CSF must pass along the sides and ventral surface of the medulla and pons and then through the tentorial incisura or opening that surrounds the midbrain. Some CSF exits the posterior fossa ventrally, along the sides of the basilar artery rostrally, and through a CSF cistern between the midbrain peduncles (i.e., basilar cistern). The major part of the flow continues along the sides of the hypothalamus to the region of the optic chiasm and then laterally and superiorly along the lateral fissure and over the parietal cortex to the superior sagittal sinus region. Here, the waterproof arachnoid has protuberances, the arachnoid villi, that penetrate the inner dura and venous walls of the superior sagittal sinus (Porth, 2008). BLOODBRAIN AND CSFBRAIN BARRIERS Maintenance of a chemically stable environment is essential to the function of the brain. In most regions of the body, extracellular fluid undergoes small fluctuations in pH and concentrations of hormones, amino acids, and potassium ions during routine daily activities such as eating and exercising. If the brain were to undergo such fluctuations, the result would be uncontrolled neural activity because some substances such as amino acids act as neurotransmitters, and ions such as K+ influence the threshold for neural firing. Two barriers, the bloodbrain barrier and the CSFbrain barrier, provide the means for maintaining the stable chemical environment of the brain. Only H2O, CO2, and O2 enter the brain with relative ease; the transport of other substances between the brain and the blood is slow (Porth, 2008). BloodBrain Barrier The bloodbrain barrier depends on the unique characteristics of the brain capillaries. Endothelial cells of brain capillaries are joined by continuous tight junctions. In addition, most brain capillaries are

surrounded by a basement membrane and by the processes of supporting cells of the brain, called astrocytes. The bloodbrain barrier permits passage of essential substances while excluding unwanted materials. Reverse transport systems remove materials from the brain. Large molecules such as proteins and peptides are largely excluded from crossing the bloodbrain barrier. Acute cerebral lesions, such as trauma and infection, increase the permeability of the blood brain barrier and alter brain concentrations of proteins, H2O, and electrolytes (Porth, 2008). Question No. 10 The bloodbrain barrier is a semipermeable component. Identify the substances that are excluded from the brain and the substances that easily cross. How is this control system or regulation made possible? Explain. Cerebrospinal FluidBrain Barrier and ArachnoidCerebrospinal Fluid Barrier The ependymal cells covering the choroid plexus are linked together by tight junctions, forming a bloodCSF barrier to diffusion of many molecules from the blood plasma of choroid plexus capillaries to the CSF. Water is transported through the choroid epithelial cells by osmosis. Oxygen and CO2 move into the CSF by diffusion, resulting in partial pressures roughly equal to those of plasma. The high sodium and low potassium contents of the CSF are actively regulated and kept relatively constant. Lipids and nonpeptide hormones diffuse through the barrier rather easily, but most large molecules, such as proteins, peptides, many antibiotics, and other medications, do not normally get through. The choroid epithelium uses energy in the form of adenosine triphosphate (ATP) to secrete actively many components into the CSF, including proteins, Na+ ions, and a number of micronutrients, such as vitamins C and B6 (pyridoxine) and folate. Because the resultant CSF has a relatively high sodium content, the negatively charged chloride and bicarbonate diffuse into the CSF along an ionic gradient. The choroid cells also generate bicarbonate from carbon dioxide in the blood. This bicarbonate is important to the regulation of the pH of the CSF. Arachnoid barrier cells prevent the CSF from contacting the blood-filled dural sinuses. This barrier essentially covers the arachnoid villi that are the structures involved in recirculating CSF back into the

blood vascular system. Mechanisms exist that facilitate the transport of other molecules such as glucose without energy expenditure. Ammonia, a toxic metabolite of neuronal activity, is converted to glutamine by astrocytes. Glutamine moves by facilitated diffusion through the choroid epithelium into the plasma. This exemplifies a major function of the CSF, that of providing a means of removal of toxic waste products from the CNS. Because the brain and spinal cord have no lymphatic channels, the CSF serves this function. Several specific areas of the brain do not have a bloodCSF barrier. One area is at the caudal end of the fourth ventricle (i.e., area postrema), where specialized receptors for the carbon dioxide level of the CSF influence respiratory function (Porth, 2008). The Autonomic Nervous System The ability to maintain homeostasis and perform the activities of daily living in an ever-changing physical environment is largely vested in the autonomic nervous system (ANS). This portion of the nervous system functions at the subconscious level and is involved in regulating, adjusting, and coordinating vital visceral functions such as blood pressure and blood flow, body temperature, respiration, digestion, metabolism, and elimination. The ANS is strongly affected by emotional influences and is involved in many of the expressive aspects of behavior. Blushing, pallor, palpitations of the heart, clammy hands, and dry mouth are several emotional expressions mediated through the ANS. Biofeedback and relaxation exercises have been used for modifying the subconscious functions of the ANS. As with the somatic nervous system, the ANS is represented in both the CNS and the PNS (Porth, 2008). Traditionally, the ANS has been defined as a general efferent system innervating visceral organs. The efferent outflow from the ANS has two divisions: the sympathetic nervous system and the parasympathetic nervous system. The afferent input to the ANS is provided by visceral afferent neurons, usually not considered a part of the ANS. The functions of the sympathetic nervous system include maintaining body temperature and adjusting blood flow and blood pressure to meet the changing needs of the body that occur with activities of daily living, such as moving from the supine to the standing position. The sympathoadrenal system also can discharge as

a unit when there is a critical threat to the integrity of the individualthe so-called fight-or-flight response. During a stress situation, the heart rate accelerates; blood pressure rises; blood flow shifts from the skin and gastrointestinal tract to the skeletal muscles and brain; blood sugar increases; the bronchioles and pupils dilate; the sphincters of the stomach and intestine and the internal sphincter of the urethra constrict; and the rate of secretion of exocrine glands involved in digestion diminishes (Porth, 2008). In contrast to the sympathetic nervous system, the functions of the parasympathetic nervous system are concerned with conservation of energy, resource replenishment and storage (i.e., anabolism), and maintenance of organ function during periods of minimal activity. The parasympathetic nervous system slows heart rate, stimulates gastrointestinal function and related glandular secretion, promotes bowel and bladder elimination, and constricts the pupil, protecting the retina from excessive light during periods when visual function is not vital to survival (Porth, 2008). The two divisions of the ANS are usually viewed as having opposite and antagonistic actions (i.e., if one activates, the other inhibits a function). Exceptions are functions, such as sweating and regulation of arteriolar blood vessel diameter, controlled by a single division of the ANS, in this case the sympathetic nervous system. The sympathetic and parasympathetic nervous systems are continually active. The effect of this continual or basal (baseline) activity is referred to as tone. The tone of an effector organ or system can be increased or decreased and is usually regulated by a single division of the ANS. For example, vascular smooth muscle tone is controlled by the sympathetic nervous system. Increased sympathetic activity produces local vasoconstriction from increased vascular smooth muscle tone, and decreased activity results in vasodilatation due to decreased tone. In structures such as the sinoatrial node and atrioventricular node of the heart, which are innervated by both divisions of the ANS, one division predominates in controlling tone. In this case, the tonically active parasympathetic nervous system exerts a constraining or braking effect on heart rate, and when parasympathetic outflow is withdrawn, the heart rate increases. The increase in heart rate that occurs with vagal withdrawal can be further augmented by sympathetic stimulation (Porth, 2008). AUTONOMIC EFFERENT PATHWAYS

The outflow of both divisions of the ANS follows a two neuron pathway. The first motoneuron, called the preganglionic neuron, lies in the intermediolateral cell column in the ventral horn of the spinal cord or its equivalent location in the brain stem. The second motoneuron, called the postganglionic neuron, synapses with a preganglionic neuron in an autonomic ganglion in the PNS (Porth, 2008). The two divisions of the ANS differ as to location of preganglionic cell bodies, relative length of preganglionic fibers, general function, nature of peripheral responses, and preganglionic and postganglionic neuromediators. This two-neuron outflow pathway and the interneurons in the autonomic ganglia that add further modulation to ANS function are features distinctly different from the arrangement in somatic motor innervation. Most visceral organs are innervated by both sympathetic and parasympathetic fibers (Porth, 2008). Sympathetic Nervous System The neurons of the sympathetic nervous system are located primarily in the thoracic and upper lumbar segments (T1 to L2) of the spinal cord; hence, the sympathetic nervous system is often called the thoracolumbar division of the ANS. These preganglionic neurons, located primarily in the ventral horn intermediolateral cell column, have axons that are largely myelinated and relatively short. The postganglionic neurons of the sympathetic nervous system are located in the paravertebral ganglia of the sympatheticchain of ganglia that lie on either side of the vertebral column, or in prevertebral sympathetic ganglia such as the celiac ganglia (Porth, 2008). Besides postganglionic efferent neurons, the sympathetic ganglia contain neurons of the internuncial, short-axon type, similar to those associated with complex circuitry in the brain and spinal cord. Many of these inhibit and others modulate preganglionic-topostganglionic transmission. The full significance of these modulating circuits awaits further investigation. The axons of the preganglionic neurons leave the spinal cord through the ventral root of the spinal nerves (T1 to L2), enter the ventral primary rami, and leave the spinal nerve through white rami of the rami communicantes to reach the paravertebral ganglionic chain (Porth, 2008). In the sympathetic chain of ganglia, preganglionic fibers may synapse with neurons of the ganglion they enter, pass up or down the

chain and synapse with one or more ganglia, or pass through the chain and move outward through a splanchnic nerve to terminate in prevertebral ganglia (i.e., celiac, superior mesenteric, or inferior mesenteric) scattered along the dorsal aorta and its branches (Porth, 2008). Preganglionic fibers from the thoracic segments of the cord pass upward to form the cervical chain connecting the inferior, middle, and superior cervical sympathetic ganglia with the rest of the sympathetic chain at lower levels (Porth, 2008). Postganglionic sympathetic axons of the cervical and lower lumbosacral chain ganglia spread further through nerve plexuses along continuations of the great arteries. Cranial structures, particularly blood vessels, are innervated by the spread of postganglionic axons along the external and internal carotid arteries into the face and the cranial cavity. The sympathetic fibers from T1 usually continue up the sympathetic chain into the head; those from T2 pass into the neck; those from T1 to T5 travel to the heart; those from T3, T4, T5, and T6 proceed to the thoracic viscera; those from T7, T8, T9, T10, and T11 pass to the abdominal viscera; and those from T12, L1, and L2 pass to the kidneys and pelvic organs. Many preganglionic fibers from the fifth to the last thoracolumbar segment pass through the paravertebral ganglia to continue as the splanchnic nerves. Most of these fibers do not synapse until they reach the celiac or superior mesenteric ganglion; others pass to the adrenal medulla (Porth, 2008). The adrenal medulla, which is part of the sympathetic nervous system, contains postganglionic sympathetic neurons that secrete sympathetic neurotransmitters directly into the bloodstream. Some postganglionic fibers, all of which are unmyelinated, exit the paravertebral ganglionic chain and reenter the segmental nerve through unmyelinated branches, called gray rami. These segmental nerves are then distributed to all parts of the body wall in the spinal nerve branches. These fibers innervate the sweat glands, piloerector muscles of the hair follicles, all blood vessels of the skin and skeletal muscles, and the CNS itself (Porth, 2008). Parasympathetic Nervous System The preganglionic fibers of the parasympathetic nervous system, also called the craniosacral division of the ANS, originate in some segments of the brain stem and sacral segments of the spinal cord.

The central regions of origin are the midbrain, pons, medulla oblongata, and sacral part of the spinal cord. The midbrain outflow passes through the oculomotor nerve (cranial nerve III) to the ciliary ganglion that lies in the orbit behind the eye; it supplies the pupillary sphincter muscle of each eye and the ciliary muscles that control lens thickness for accommodation. From the caudal pontine outflow originate the preganglionic fibers of the intermedius component of the facial nerve (cranial nerve VII) complex. This outflow synapses in the submandibular ganglia, which sends postganglionic fibers to supply the submandibular and sublingual glands. In addition, preganglionic fibers are distributed to the pterygopalatine ganglia to synapse on postganglionic neurons. These postganglionic fibers emanating from the pterygopalatine ganglia supply the lacrimal and nasal glands. The medullary outflow develops from cranial nerves VII, IX, and X. Fibers in the glossopharyngeal nerve (cranial nerve IX) synapse in the otic ganglia, which supply the parotid salivary glands (Porth, 2008). CENTRAL INTEGRATIVE PATHWAYS General visceral afferent fibers accompany the sympathetic and parasympathetic outflow into the spinal and cranial nerves, bringing chemoreceptor, pressure, organ capsule stretch, and nociceptive information from organs of the viscera to the brain stem, thoracolumbar cord, and sacral cord (Porth, 2008). Local reflex circuits relating visceral afferent and autonomic efferent activity are integrated into a hierarchic control system in the spinal cord and brain stem. Progressively greater complexity in the responses and greater precision in their control occur at each higher level of the nervous system. Most visceral reflexes contain contributions from the LMNs that innervate skeletal muscles as part of their response patterns (Porth, 2008). The distinction between purely visceral and somatic reflex hierarchies becomes less and less meaningful at the higher levels of hierarchic control and behavioral integration. For most autonomicmediated functions, the hypothalamus serves as the major control center. The hypothalamus, having connections with the cerebral cortex, the limbic system, and the pituitary gland, is in a prime position to receive, integrate, and transmit information to other areas of the nervous system (Porth, 2008).

Neurons concerned with thermoregulation, thirst, and feeding behaviors are also found in the hypothalamus. The hypothalamus also is the site for integrating neuroendocrine function. Hypothalamic releasing and inhibiting hormones control the secretion of anterior pituitary hormones (i.e., thyroidstimulating hormone, corticotropin, growth hormone, luteinizing hormone, follicle-stimulating hormone, and prolactin). The supraoptic nuclei of the hypothalamus are involved in water metabolism through synthesis of antidiuretic hormone and its release from the posterior pituitary gland. Oxytocin, which causes contraction of the pregnant uterus and milk letdown during breast feeding, is synthesized in the hypothalamus and released from the posterior pituitary gland similar to that of antidiuretic hormone (Porth, 2008). AUTONOMIC NEUROTRANSMISSION The generation and transmission of impulses in the ANS occur in the same manner as in other neurons. There are self-propagating action potentials with transmission of impulses across synapses and other tissue junctions by way of neurohumoral transmitters. However, the somatic motoneurons that innervate skeletal muscles divide into many branches, with each branch innervating a single muscle fiber; in contrast, the distribution of postganglionic fibers of the ANS forms a diffuse neural plexus at the site of innervations (Porth, 2008). The membranes of the cells of many smooth muscle fibers are connected by conductive protoplasmic bridges, called gap junctions, that permit rapid conduction of impulses through whole sheets of smooth muscle, often in repeating waves of contraction. Autonomic neurotransmitters released near a limited portion of these fibers provide a modulating function extending to many effector cells. Smooth muscle layers of the gut and of the bladder wall are examples. Sometimes, isolated smooth muscle cells are individually innervated by the ANS, such as the piloerector cells that elevate the hair on the skin during cold exposure (Porth, 2008). The main neurotransmitters of the autonomic nervous system are acetylcholine and the catecholamines, epinephrine and norepinephrine. Acetylcholine is released at all preganglionic synapses in the autonomic ganglia of both sympathetic and parasympathetic nerve fibers and from postganglionic synapses of all parasympathetic nerve endings. It also is released at sympathetic nerve endings that

innervate the sweat glands and cholinergic vasodilator fibers found in skeletal muscle (Porth, 2008). Norepinephrine is released at most sympathetic nerve endings. The adrenal medulla, which is a modified prevertebral sympathetic ganglion, produces epinephrine along with small amounts of norepinephrine. Dopamine, which is an intermediate compound in the synthesis of norepinephrine, also acts as a neurotransmitter. It is the principal inhibitory transmitter of internuncial neurons in the sympathetic ganglia. It also has vasodilator effects on renal, splanchnic, and coronary blood vessels when given intravenously and is sometimes used in the treatment of shock (Porth, 2008). Many neurons secreting peptide molecules have been identified in ANS ganglia, especially in the enteric plexus and in postganglionic ANS terminals of the sympathetic and parasympathetic systems. Many of these are secreted by internuncial neurons or as additional transmitters or cotransmitters by preganglionic and postganglionic neurons. Binding at postsynaptic neuropeptide receptors usually does not result in action potentials; instead, it alters the membrane potential or receptor numbers, producing long-term (minutes to hours) changes in responsiveness to the neurotransmitter. For example, dual secretion of norepinephrine and neuropeptide Y in some sympathetic vasoconstrictor terminals results in longer vasomotor constriction (Porth, 2008). Acetylcholine and Cholinergic Receptors Acetylcholine is synthesized in the cholinergic neurons from choline and acetyl coenzyme A (acetyl CoA). After acetylcholine is secreted by the cholinergic nerve endings, it is rapidly broken down by the enzyme acetylcholinesterase. The choline molecule is transported back into the nerve ending, where it is used again in the synthesis of acetylcholine. Receptors that respond to acetylcholine are called cholinergic receptors (Porth, 2008). Two types of cholinergic receptors are known: muscarinic and nicotinic. Muscarinic receptors are present on the innervational targets of postganglionic fibers of the parasympathetic nervous system and the sweat glands, which are innervated by the sympathetic nervous system. Nicotinic receptors are found in autonomic ganglia and the end plates of skeletal muscle. Acetylcholine is excitatory to most muscarinic and nicotinic receptors, except those in the heart and lower esophagus, where it has an inhibitory effect. The drug atropine

is an antimuscarinic or muscarinic cholinergic-blocking drug that prevents the action of acetylcholine at excitatory and inhibitory muscarinic receptor sites. Because it is a muscarinic-blocking drug, it exerts little effect at nicotinic receptor sites (Porth, 2008). Catecholamines and Adrenergic Receptors The catecholamines, which include norepinephrine, epinephrine, and dopamine, are synthesized in the axoplasm of sympathetic nerve terminal endings from the amino acid tyrosine. During catecholamine synthesis, tyrosine is hydroxylated (i.e., has a hydroxyl group added) to form DOPA, and DOPA is decarboxylated (i.e., has a carboxyl group removed) to form dopamine. Dopamine in turn is hydroxylated to form norepinephrine. In the adrenal gland, an additional step occurs during which norepinephrine is methylated (i.e., a methyl group is added) to form epinephrine (Porth, 2008). Each step in sympathetic neurotransmitter synthesis requires a different enzyme, and the type of neurotransmitter produced depends on the types of enzymes that are available in a nerve terminal. For example, the postganglionic sympathetic neurons that supply blood vessels synthesize norepinephrine, but postganglionic neurons in the adrenal medulla produce epinephrine or norepinephrine. Epinephrine accounts for approximately 80% of the catecholamines released from the adrenal gland. The synthesis of epinephrine by the adrenal medulla is influenced by the glucocorticoid secretion from the adrenal cortex (Porth, 2008). Some drugs, such as the tricyclic antidepressants, are thought to increase the level of catecholamines at the site of nerve endings in the brain by blocking the reuptake process. Others, such as the MAO inhibitors, decrease the enzymatic degradation of the neurotransmitters and increase their levels (Porth, 2008). Catecholamines can cause excitation or inhibition of smooth muscle contraction, depending on the site, dose, and type of receptor present. Norepinephrine has potent excitatory activity and low inhibitory activity. Epinephrine is potent as both an excitatory and an inhibitory agent. The excitatory or inhibitory responses of organs to sympathetic neurotransmitters are mediated by interaction with special structures in the cell membrane called receptors (Porth, 2008).

Suggested Reading Activity Porth, Carol Mattson. 2008. Pathophysiology: Concepts of Altered Health States. 7th Edition. Philadelphia, USA: Lippincott Williams and Wilkins Bullock, Barbara L. and Henze, Reet L. 2000. Focus on Pathophysiology. Philadelphia, USA: Lippincott Williams & Wilkins.

Instructional References: Adams, D.A., Victor, M., and Hopper, A.H. (1998). Principles of Neurology. 6th Ed. New York,USA: McGraw-Hill. Bullock, Barbara L. and Henze, Reet L. 2000. Focus on Pathophysiology. Philadelphia, USA: Lippincott Williams & Wilkins. Martini, Frederic H. 2002. Fundamentals of Anatomy and Physiology. 6th Edition. New York, New York: McGraw-Hill Companies, Inc. Snell, R.S. 1997. Clinical Neuroanatomy for Medical Students. 4th Edition. Philadelphia, USA: Lippincott-Raven

Unit III

Alterations in Mobility Module 1 The Skeletal System Overview The human form is distinctive and unique in many respects. A human has eyes at the front of a head roughly the shape of a bowling ball, balanced above the trunk on a relatively slim and delicate neck that lets the head rotate and bend to bring objects into clearer view. The body stands on two relatively long lower limbs, freeing the upper limbs for grasping or manipulating things in the environment. The extra height also gives the eyes a greater field of vision, but it makes balance more of a problem than it is for mammals standing on four legs. Head, trunk, and limbs, balance, and manipulationthese features are totally dependent on the support provided by the skeletal system. Movement and the maintenance of an upright posture represent the action of the skeleton and muscle groups. There are 340 skeletal muscles in the body, found in pairs on the right and left side (Bullock and Henze, 2000).

Objectives After you have completed this module, you should be able to: 1. discuss the principal functions of the skeleton and identify the body systems served by these functions 2. list the different kinds of bone cells, describe their functions, and discuss the organization of bone tissue 3. compare and contrast the three principal categories of joints, using specific joints as examples 4. describe what a

Coordination of skeletal bones and muscles provides humans with functional abilities ranging from gross motor activities to fine, precise mobility. From the moment of conception, the human body is programmed to perform with coordination for many years. When alterations in musculoskeletal function occur, the entire body is affected (Bullock and Henze, 2000).

Suggested Reading Activity

Martini, Frederic H. 2002. Fundamentals of Anatomy and Physiology. 6th Edition. New York, New York: McGraw-Hill Companies, Inc. Chapter 6, pp. 183-284 Bone Histology Osseous tissue is a supporting connective tissue. Like other connective tissues, osseous tissue contains specialized cells and a matrix consisting of extra cellular protein fibers and a ground substance. The matrix of bone tissue is solid and sturdy, owing to the deposition of calcium salts around the protein fibers (Martini, 2002). The following are characteristics of bone: 1. The matrix of bone is very dense and contains deposits of calcium sats.
2. The matrix contains bone cells, or osteocytes, within pockets

called lacunae.
3. Canaliculi, narrow passageways through the matrix, extend

between the lacunae and nearby blood vessels, forming a branching network for the exchange of nutrients, waste products, and gases.
4. Except at joints, the outer surfaces of bones are covered by a

periosteum, which consists of outer fibrous and inner cellular layers. Cells in Bone 1. Osteocytes 2. Osteoblasts 3. Osteoprogenitor cells The Skeleton As A Calcium Reserve The bones of the skeleton are important reservoirs. Calcium is the most abundant mineral in the human body. A typical human body contains 1-2 kg (2.2-4.4 lb.) of calcium, with roughly 99% of it deposited in the skeleton.

Calcium ions play a role in a variety of physiological processes, so the body must tightly control calcium ion concentrations in order to prevent damage to essential physiological systems. Even small variations from the normal concentration will affect cellular operations. Larger changes can cause a clinical crisis. If calcium levels decrease by 35%, neurons become so excitable that convulsions can occur. A 50% reduction in calcium concentration generally causes death. Hormones and Calcium Balance Calcium ion homeostasis is maintained by a negative feedback system involving a pair of hormones with opposing effects. These hormones, parathyroid and calcitonin, coordinate the storage, absorption, and excretion of calcium ions. The three target sites are involved: (1) the bones (storage), (2) the digestive tract (absorption), (3) the kidneys (excretion). When calcium ion concentrations in the blood fall below normal, cells of the parathyroid glands, embedded in the thyroid gland in the neck, release parathyroid hormone (PTH) into the bloodstream. Question No. 11 What effect would increased PTH secretion have on blood calcium levels? Explain the homeostatic feedback mechanism. If the calcium ion concentration of the blood instead rises above normal, special cells (parafollicular cells, or C cells) in the thyroid gland secrete calcitonin. Question No. 12 How does calcitonin help lower calcium ion concentration of the blood? Explain the homeostatic feedback mechanism.

Question No. 3 Why does a child who has rickets have difficulty walking? Fracture Repair Despite its mineral strength, bone can crack or even break if subjected to extreme loads, sudden impacts, or stresses from unusual directions. The damage produced constitutes a fracture. Most

fractures heal even after severe damage, provided that the blood supply and the cellular components of the endosteum and periosteum survive. The steps in the repair of fracture involves the formation of a fracture hematoma, an external callus, and an internal callus. As the repair continues, osteoblasts replace the central cartilage of the external callus with spongy bone. Osteoclasts and osteoblasts continue to remodel the region of the fracture for a period ranging from four months to well over a year. Aging and The Skeletal System The bones of the skeleton become thinner and weaker as a normal part of the aging process. Inadequate ossification is called osteopenia, and all of us become slightly osteopenic as we age. This reduction in bone mass begins between the ages of 30 and 40. Over that period, osteoblast activity begins to decline, while osteoclast activity continues at previous levels. Once the reduction begins, women lose roughly 8% of their skeletal mass every decade, whereas the skeletons of men deteriorate at about 3% per decade. Not all parts of the skeleton are equally affected. Epiphyses, vertebrae, and the jaws lose more than their share, resulting in fragile limbs, a reduction in height, and the loss of teeth. When the reduction in the bone mass is sufficient to compromise normal function, the condition is known as osteoporosis. The fragile bones that result are likely to break when exposed to stresses that younger individuals could easily tolerate. For example, a hip fracture can occur when a woman in her nineties simply tries to stand. Any fractures that do occur lead to a loss of independence and an immobility that further weakens the skeleton. Sex hormones are important in maintaining normal rates of bone deposition. Over age 45, an estimated 29% of women and 18% of men have osteoporosis. Question No. 14 Why is osteoporosis more common in older women than in older men? Explain the pathogenesis of this condition. Articulations Two classification methods are used to categorize joints. The first classification is based on the range of motion permitted: 1. Synarthrosis - is an immovable joint. This joint can be fibrous or cartilaginous, depending on the

nature of the connection. 2. Amphiarthrosis - is a slightly movable joint. This is also fibrous or cartilaginous, depending on the nature of the connection between the opposing bones. 3. Diarthrosis - is a freely movable joint. This is a synovial joint. Synarthroses (Immovable Joints): 1. Sutures 2. Gomphoses 3. Synchondroses 4.Synostoses Amphiarthroses (Slightly Movable Joints): 1. Syndesmosis 2. Symphysis Diarthrosis (Freely Movable Joints): Objectives 1. Synovial joints After you have completed this module, Unit IV you should be able to: Alterations in Mobility 1. Specify the functions Module II of The Muscular System skeletal muscle tissue 2. Describe the organization of muscle at the tissue level 3. Explain the unique characteristics of skeletal muscle fibers 4. Identify the components of the neuromuscular junction, and summarize the events involved in the neural control of skeletal muscles 5. Describe the mechanism responsible for tension production in a muscle fiber, and Overview Think for a moment what life would be like without muscle tissue. Imagine being unable to sit, stand, walk, speak, or grasp objects. Imagine, too, how your internal functions would be affected. Blood would not circulate, because you would have no heartbeat to propel it through the vessels. You would be unable to breathe, speak, or eat, and food could not move along your gastrointestinal tract. In fact, there would be practically no movement along

any of your (Martini, 2002).

internal

passageways

This is not to say that all life depends on muscle tissue. Some large organisms get by very nicely without it; we call them plants. But life as we live it would be impossible, because virtually all of our dynamic interactions with the environment involve muscle tissue (Martini, 2002). The muscular system includes all the skeletal muscles that can be controlled voluntarily. Most of the muscle tissue in the body is part of this system, and approximately 700 skeletal muscles have been identified. Many professions, from dancing to coaching to physical therapy, rely on an understanding of the functional anatomy of skeletal muscles.

Without the skeletal system, movement in the external environment would not be possible. The bones of the skeletal system serve as a framework for the attachment of muscles, tendons, and ligaments. The skeletal system protects and maintains soft tissues in their proper position, provides stability for the body, and maintains the bodys shape. The bones act as a storage reservoir for calcium, and the central cavity of some bones contains the hematopoietic connective tissue in which blood cells are formed (Porth, 2008).

Suggested Reading Activity Marieb,Elaine N. 2006. Anatomy and Physiology. 8th Edition. Jurong, Singapore: Pearson Education, Inc. Martini, Frederic H. 2002. Fundamentals of Anatomy and Physiology. 6th Edition. New York, New York: McGraw-Hill Companies, Inc. Van De Graaff, Kent M. et. al. 1997. Synopsis of Human Anatomy and Physiology. Boston, Massachusetts, USA: WCB/McGraw-Hill

Normal Structure of Skeletal Muscles Skeletal or striated muscle forms the voluntary muscular system. Skeletal muscle is the predominant type of muscle in the human body and composes approximately 40%-50% of the adult body weight. Muscles are arranged in bundles encased in a sheath of connective tissue called the epimysium. The epimysium binds muscle to one another and to ligaments and bones. Muscle Fibers Striated muscle fibers differ from cardiac and visceral (smooth) muscle fibers. Whereas striated and cardiac muscle have similar appearance, with the exception of the placement of the nucleus, smooth muscle lacks the characteristic striations. Smooth muscles are composed of many fibrils and divided into multiunit and visceral muscles. Multiunit smooth muscles are stimulated mainly by nerve signals such as those causing contraction of blood vessels. Visceral smooth muscles are arranged in contacting sheets or bundles, which allows them to contract as a group. This type of smooth muscle is found in many organs, including the gastrointestinal system. A single muscle is composed of many muscle fibers or myofibers. Muscle fibers are muscle cells consisting of bundles of long, multinucleated cells in which the oval-shaped nuclei are close to the cell membrane. Each muscle fiber is bound by a network of delicate tissue called endomysiyum. This tissue contains an extensive supply of capillaries and nerve fibers and provides support for the blood vessels and nerves that are adjacent to the muscle fibers. Muscles have a reddish appearance due to the presence of the myoglobin pigment, an O2 depot in muscles. The muscles that must

maintain activity for periods of time usually contain more myoglobin than others. Muscle fibers are arranged in bundles called fascicles, which are embedded in a web of connective tissue called the perimysium. Many fascicles constitute a single muscle, and these are encased in the epimysium, which tapers into the tendon that connects to the bone. Nerve Supply to Muscles The nerve supply to large skeletal muscle fibers (extrafusal) stems through large A alpha fibers originating from alpha motor neurons. The nerve supply to small skeletal muscle fibers (intrafusal) in muscle spindles originate through A gamma fibers from small gamma motor neurons. Fibers from both large and small nerves transmit together from the spinal cord via the anterior horn (anterior motor neuron). A single motor neuron may innervate many muscle fibers. All muscle fibers innervated by a single neuron constitute one motor unit, the functional unit of skeletal muscle. All muscle fibers in a single motor unit are of the same type and vary greatly in number, depending on the nature of the movement. Neuron fibers innervating muscles more resistant to fatigue and requiring less precise movement, such as those in the leg calf, innervate large numbers of muscle fibers. However, neurons innervating muscles that fatigue more easily and require more precise movement, such as those of the eye of hand, innervate far fewer muscle fibers. Central control over the motor neurons is achieved by means of multiple descending direct and indirect pathways, including the principal efferent motor fibers of the corticospinal tract. The innervations from these usually comes through multiple spinal interneurons. In its simplest form, neuronal innervation at the spinal level is the monosynaptic muscle stretch reflex. The afferent fiber to the dorsal horn from a muscle spindle synapses within the cord with an alpha motor fiber that returns innervations via the anterior horn to the same muscle spindle.

Question No. 15

Describe what happens to muscles that are not used on a regular basis. What can be done to offset this effect?

Neuromuscular Junction (Myoneural Junction) The junction between a motor nerve ending and a muscle fiber is called a neuromuscular junction or myoneural junction. Each skeletal muscle fiber is contacted by at least one nerve ending. One motor nerve fiber can stimulate several muscle fibers at the same time. The axon terminals form flattened motor end plates on the surface of the muscle fibers. The end branches of the motor neuron, known as axon terminals, gain access to the muscle fiber through the endomysium. At the junction between the muscle fiber and the motor neuron, the muscle fiber membrane forms a motor end plate. Suggested Reading Activity Bullock, Barbara L. and Henze, Reet L. 2000. Focus on Pathophysiology. Philadelphia, USA: Lippincott Williams & Wilkins.

Question No. 16 Atracurium is a drug that blocks the binding of Acsh to receptors. Give an example of a site where such binding normally occurs, and predict the physical effect of this drug.

Muscle Tension The basic response of muscle to a single stimulus is a twitch. When stimulated, the fiber develops tension rapidly and then relaxes. Tension can be increased by more rapid stimulation. The twitch tension for each higher stimulation rate increases in a steplike manner until tetanus (not the disorder), which is the full and complete contraction

of the muscle. Muscle tension can be altered by increasing the rate of stimulation. The force of muscle contraction can be increased by the incremental recruitment of additional motor units by the central nervous system. Suggested Reading Activity Martini, Frederic H. 2002. Fundamentals of Anatomy and Physiology. 6th Edition. New York, New York: McGraw-Hill Companies, Inc.

Question No. 17 Why does a client diagnosed to have infection with Clostridium tetani exhibit muscle stiffness and difficulty in swallowing? Instructional References: Bullock, Barbara L. and Henze, Reet L. 2000. Focus on Pathophysiology. Philadelphia, USA: Lippincott Williams & Wilkins. Martini, Frederic H. 2002. Fundamentals of Anatomy and Physiology. 6th Edition. New York, New York: McGraw-Hill Companies, Inc. Porth, Carol Mattson. 2005. Pathophysiology Concepts of Altered Health States. Philadelphia, USA: Lippincott Williams and Wilkins Seeley, Rod R. et. al. 2006. Anatomy and Physiology. 7th Edition. New York, New York: McGraw-Hill Companies, Inc. ------------------------------------END----------------------------------

SOUTHWESTERN UNIVERSITY Cebu City GRADUATE SCHOOL OF HEALTH SCIENCE, MANAGEMENT, AND PEDAGOGY REQUIREMENT IN MSNG 201 (Advanced MSN 1) Name: ______________________ ___________________ Professor: Odilon A. Maglasang,RN,MAN 7:00 Date: Schedule: Saturday,1:00-

(Technicalities: Margin is one inch in all sides. Font style is Verdana Font size is 12)

Вам также может понравиться